Review Pathology
Review Pathology
Review Pathology
Pathology
NOTICE
Medicine is an ever-changing science. As new research and clinical
experience broaden our knowledge, changes in treatment and drug therapy
are required. The authors and the publish- er of this work have checked with
sources believed to be reliable in their efforts to provide information that is
complete and generally in accord with the standards accepted at the time of
publication. However, in view of the possibility of human error or changes
in medical sciences, neither the authors nor the publisher nor any other party
who has been involved in the preparation or publication of this work
warrants that the information contained herein is in every respect accurate or
complete, and they disclaim all responsibility for any errors or omissions or
for the results obtained from use of the information contained in this work.
Readers are encouraged to confirm the information contained herein with
other sources. For example and in particular, readers are advised to check the
product information sheet in- cluded in the package of each drug they plan to
administer to be certain that the information contained in this work is
accurate and that changes have not been made in the recommended dose or in
the contraindications for administration. This recommendation is of particular
im- portance in connection with new or infrequently used drugs.
DEJA REVIEW™
Pathology
Second Edition
Jessica L. Davis, MD
ISBN: 978-0-07-173677-0
MHID: 0-07-173677-8
The material in this eBook also appears in the print version of this title:
ISBN: 978-0-07-162714-6, MHID: 0-07-162714-6.
All trademarks are trademarks of their respective owners. Rather than put a
trademark sym- bol after every occurrence of a trademarked name, we use
names in an editorial fashion only, and to the benefit of the trademark owner,
with no intention of infringement of the trade- mark. Where such
designations appear in this book, they have been printed with initial caps.
TERMS OF USE
Preface
Acknowledgments
Chapter 2 BIOCHEMISTRY
General Principles
DNA/RNA/Protein
Pathology Nutritional
Disorders Vitamins and
Minerals
Poisoning/Chemical
Injury Adverse Effects of
Drugs Biochemical
Diseases Clinical
Vignettes
Chapter 12 NEUROPATHOLOGY
Embryology
Anatomy
Histology
Neuropatholog
y Clinical
Vignettes
Chapter 13 DERMATOPATHOLOGY
Embryology
Anatomy/Histol
ogy Pathology
Clinical Vignettes
Index
Faculty Reviewer
Student Reviewers
Pete
Pelletier,
M(ASCP)
University
of Utah
School of
Medicine
Class of
2010
Sheree Perron
Eastern Virginia
Medical School Class
of 2010
Robert Nastasi
SUNY Upstate
Medical University
Class of 2009
Preface
The Déjà Review series is a unique resource that has been designed to allow
you to review the essential facts and determine your level of knowledge on
subjects tested on Step 1 of the United States Medical Licensing
Examination (USMLE). This second edition of Déjà Review: Pathology is
designed for the students as a compact yet high-yield review of ma- jor
pathophysiologic and histopathologic concepts which make up a large
percentage of USMLE Step 1 questions and which will contribute to overall
mastery of this subject matter.
ORGANIZATION
This text was assembled with the intent to represent the core topics tested
on course exam- inations and USMLE Step 1. Remember, this text is not
intended to replace comprehensive textbooks, course packs, or lectures. It is
simply intended to serve as a supplement to your studies during your
pathophysiology and histopathology course work and Step 1 prepara- tion.
You may use the book to quiz yourself or classmates on topics covered in
recent lec- tures and clinical case discussions. A bookmark is included so
that you can easily cover up the answers as you work through each chapter.
The compact, condensed design of the book facilitates portability and will
allow you to review this material practically anywhere you wish.
However you choose to study and whatever your learning style, we hope
you find this re- source helpful throughout your preparation for course
examinations and the USMLE Step 1.
Jessica and Emily
Acknowledgments
We would like to acknowledge Dr Terry Morgan for his advice and support.
We would also like to acknowledge the faculty in the Department of
Pathology at Oregon Health & Science University whose enthusiasm for
pathology has encouraged and motivated us throughout medical school and
especially in the preparation of this second edition. Special thanks also to
Charles Fredman for assistance preparing the digital images featured in this
edition.
We would also like to acknowledge and give special thanks to Sarah
Galfione, Kenny Kronforst, and Julia Conlon, the authors of the first
edition of Deja Review: Pathology, for beginning this project of building a
comprehensive and thorough review of a large and com- plicated subject
matter. We hope our additions and revisions will only improve the frame-
work they created.
CHAPTER 1
General Concepts in Pathology
Define pathology:
The study of suffering (from the Greek pathos), or the study of
functional changes in cells, tissues, and organs that underlie disease
Define homeostasis:
State of internal equilibrium at which normal physiologic demands of
a cell are met; pathophysiology results when stimuli (ie, cell injury)
sufficiently disrupt homeostasis.
Define hyperplasia:
An increase in number of cells as an adaptive response to stress,
usually resulting in increased volume of an organ or tissue. Cells must
be capable of mitotic division (eg, prostate).
Define hypertrophy:
An increase in cell size due to synthesis of cellular structural
components as an ad- aptive response to stress, usually resulting in
increased size of an organ or tissue. Does not require mitotic division
(eg, myocardium).
Define atrophy:
Reduction of cell size due to loss of structural components of the
cell. An attempt by the cell to reduce demand to match reduced supply.
The entire tissue/organ dimin- ishes in size when enough cells are
involved.
What is the process of reversible change that occurs when one adult cell
type is re- placed by another adult cell type?
Metaplasia
Define karyolysis:
Dissolution of the nucleus (karyo- = nucleus, -lysis = to break apart)
Define karyorrhexis:
Nuclear fragmentation (karyo- = nucleus, -rrhexis = rupture)
Define pyknosis:
Nuclear shrinkage and condensation
1. Apoptosis
2. Necrosis
Define apoptosis:
A process of cell death by which a cell activates enzymes
(“caspases”) that de- grade the cell’s own DNA and proteins (ie,
“programmed cell death”) while maintain- ing an intact plasma
membrane. The entire cell is then phagocytized.
Define necrosis:
A process of cell death by lysosomal enzymatic digestion and loss of plasma
mem- brane integrity
What are the examples of triggers of apoptosis via the intrinsic pathway?
Lack of hormonal or growth factor stimulation, DNA damage
leading to p53 ac- tivation
What are the examples of triggers of apoptosis via the extrinsic pathway?
Tumor necrosis factor (TNF) receptor ligands (ie, TNF-α), FAS
receptor ligands (ie, FasL)
Define hypoxia:
A state of reduced oxygen availability (ie, poor hemoglobin
saturation, inadequate ventilation, hemolysis)
Define ischemia:
A state of significantly reduced blood flow (ie, thrombotic occlusion,
trauma), which leads to tissue damage if not reversed
What are the early consequences of ischemic injury?
Transient shift to anaerobic glycolysis; disturbed ionic and fluid
balance; inhibited beta-oxidation of fatty acids
What is autophagy?
Lysosomal digestion of a cell’s own components
Define jaundice:
Excess of bilirubin within cells and tissues
Define inflammation:
Biologic response to a perceived injurious agent that results in
vascular changes which allow fluid and leukocytes into extravascular
tissue.
What features and cell type typically characterize the acute phase of
inflammation?
Hyperemia (rubor), pain (dolor), heat (calor), edema/swelling
(tumor); poly- morphonuclear (PMN) leukocytes
What is edema?
Excess transudative or exudative fluid in the interstitial space or a
body cavity
Define transudate:
A clear, extravascular, low-protein, low-cellularity fluid usually due
to changes in hydrostatic or osmotic pressure. Specific gravity is <1.012.
Define exudate:
A clear to cloudy extravascular, high protein, high-cellularity fluid
usually due to changes in capillary permeability. Specific gravity is
>1.012.
What is the term for an exudate rich in neutrophils and parenchymal cell
debris?
Pus (purulent exudate)
1. Histamine
2. Serotonin
1. C3b
2. iC3b
*“Be covered”
What is the role of the kinin system in inflammation?
The kinin system serves to produce bradykinin which mediates
vascular permeab- ility and vasodilation.
1. Serous inflammation
2. Fibrinous inflammation
3. Suppurative inflammation
4. Ulcerative inflammation
What are the four possible outcomes of acute inflammation?
1. Complete resolution
2. Fibrosis
3. Abscess formation
4. Chronic inflammation
1. Continued recruitment
2. Local proliferation
3. Immobilization at the target site by cytokines and oxidized lipids
1. Foreign-body granuloma
2. Immune granuloma
Define neoplasm:
Neo- = new, -plasia = growth. This term can be applied to any
mass, benign or malignant.
Define dysplasia:
A process wherein nuclear features of malignancy are observed but
general archi- tectural features are benign
Define anaplasia:
Lack of differentiation (almost always indicative of malignancy)
What does the suffix -sarcoma indicate?
A malignant neoplasm of mesenchymal origin, including bone and
soft tissue
What is a carcinoma?
A malignant neoplasm of epithelial origin
What is a tumor that arises from germ cells and contains cells from more
than one germ layer?
Teratoma
What is the “guardian of the genome” and how do mutations affect its
function?
p53, a tumor suppressor gene which normally serves to arrest the
cell cycle and induce apoptosis under condition of DNA damage.
When mutated, this function is lost and cells with DNA damage will
continue to divide and proliferate, possibly acquiring mutations which
confer malignant behavior.
CLINICAL VIGNETTES
A 5-year-old falls and bumps his knee. Three days later, there is a blue
bruise. Why is it blue?
Lysis of erythrocytes causes hemoglobin to break down into other
pigments.
GENERAL PRINCIPLES
What is the name given to segments of DNA which are spliced out of
primary mRNA transcripts?
Introns
DNA/RNA/PROTEIN PATHOLOGY
NUTRITIONAL DISORDERS
What is marasmus?
A nutritional deficiency of both total calories and protein
What is kwashiorkor?
A nutritional protein deficiency in the setting of adequate total
calorie intake
POISONING/CHEMICAL INJURY
Which drug can cause profuse cutaneous flushing (ie, red man
syndrome)?
Vancomycin
1. Leucine
2. Lysine
What foods must be eliminated from the diet in patients with PKU?
Any foods containing phenylalanine
1. Hunter syndrome
2. Fabry disease
1. Hunter syndrome
2. Hurler syndrome
What lysosomal storage disease is characterized by a deficiency of β-
glucocerebrosi- dase?
Gaucher disease
*G auCher: problems with GC—Gathered Crinkled cells,
GlucoCerebroside in- crease, and (β-GlucoCerebrosidase deficiency
CLINICAL VIGNETTES
A 35-year-old white woman is found dead at her home. She was going
through a di- vorce and her family suspects that her husband has done
something that resulted in her death. At autopsy, the pathologist
recognizes a distinct scent of bitter almonds and notes no abnormalities
except for scattered petechial hemorrhages. What is the most likely
cause of death?
Cyanide poisoning
A family is found dead in their home after a large ice storm and power
outage. There was an old gas heater warming the home. At autopsy,
the bodies have a cherry-red coloration of the lips, viscera, and
muscles. What is the most likely cause of death?
Carbon monoxide (CO) poisoning
GENERAL PRINCIPLES
What is aneuploidy?
Possessing a chromosome number that is not a multiple of the
normal haploid num- ber, typically arising from an error in meiosis or
mitosis
What is the condition of having one or more additional sets of the haploid
number of chromosomes?
Polyploidy (eg, tetraploidy or triploidy)
What is an inversion?
Chromosomal rearrangement resulting from two break points on the
same chro- mosome, with subsequent reversal and reincorporation into
the chromosome.
What type of chromosome has either two short arms or two long arms
due to faulty separation of the centromere during meiosis?
Isochromosome
What is the process by which all but one X chromosome in each cell is
randomly inac- tivated early in embryonic development?
Lyonization
CHROMOSOMAL ABNORMALITIES
Are males with a 47,XYY karyotype easily distinguished from males with
a normal 46,XY karyotype?
No, but they are generally tall-with an increased risk of having
learning difficulties and/or behavioral problems.
What two syndromes are associated with genetic imprinting of the long
arm of chro- mosome 15?
1. Prader-Willi
2. Angelman syndrome
What is the term for having two identical alleles for a given gene?
Homozygous
What is the term for having two different alleles for a given gene?
Heterozygous
What is the term for having only one copy (allele) of a gene?
Hemizygous
What is co-dominance?
Two alleles share dominance (eg, AB blood group)
A woman with Marfan syndrome and her normal husband wish to have
children. What is the probability that their child will have Marfan
syndrome?
50%
What molecular genetic tool may be used for early diagnosis of this
disease?
PCR or DNA sequencing; previously restriction fragment length
polymorphism (RFLP) studies were used
What AD disease is the most common growth plate disorder and a major
cause of dwarf ism?
Achondroplasia
Which type of OI is the result of a null allele, and which types of OI are
the results of missense mutations?
What signs should one look for when examining a patient if von Gierke
disease is sus- pected?
Massive hepatomegaly causing a prominent abdomen; fat deposits in
cheeks and buttocks; convulsions due to hypoglycemia
• Treatment—phenylalanine-free diet
• Prognosis—good as long as screening for PKU identifies the
disease very early and the parents/patient are compliant with a
phenylalanine-free diet to prevent pro- gressive mental/neurologic
deterioration
What is ochronosis?
A term describing the pigmentation or darkening of fibrous tissue or
cartilage
X-LINKED
What X-linked recessive disorder usually presents within the first year
of life with gross motor developmental delay and hypotonia, with
physical examination showing
impaired growth and neurologic findings including hyperreflexia,
spasticity, and choreoathetosis?
Lesch-Nyhan syndrome
• Angiokeratomas
• Burning pain the extremities
• Febrile episodes
• Corneal opacities
• Cardiac complications including stroke and renal failure
A newborn baby presents with a flat nasal bridge, low-set eats, simian
crease, and ventricular septal defect. What is the most likely diagnosis?
What other features may be present that would also support the
diagnosis?
Down syndrome
Also look for upslanting epicanthal folds, wide-set eyes, and expect
development- al delay/mental retardation
A tall-statured 25-year-old man presents with hypogonadism, slight
gynecomastia, and fertility problems. A single Barr body is observed in
a buccal smear preparation. What condition is most likely?
Klinefelter syndrome, 47,XXY
BACTERIA
Which bacterium has no cell wall and commonly is associated with high
IgM titers?
Mycoplasma pneumoniae
What stain should be done to detect and help classify most bacteria?
Gram stain
What stain can help detect Legionella and fungi like Pneumocystis
jiroveci (formerly carinii)?
Silver stain
VIRUSES
What virus classically has inclusions that look like “owl’s eyes”?
Cytomegalovirus (CMV)
What is the name and location of the inclusion body seen in rabies virus
infection?
Negri bodies—eosinophilic, cytoplasmic inclusion in neurons,
most commonly in pyramidal cells of the hippocampus
What and why must a confirmatory test be done if an HIV ELISA comes
back posit- ive?
ELISA is a screening test, therefore highly sensitive but may result
in false-pos- itive test results. HIV Western blot is needed to confirm
the diagnosis because it is highly specific; therefore, a positive result
on Western blot will confirm the ELISA diagnosis and a negative
result on Western blot will identify a false-positive ELISA test.
FUNGI
1. Histoplasma
2. Blastomyces
PROTOZOA
What are the various species of that cause malaria and which is the most
virulent?
Plasmodium vivax, P. ovale, P. malariae, P. falciparum—P.
falciparum results in the most severe infection
What infection has “maltese cross” (merozoite) forms in addition to
trophozoite ring forms on peripheral blood smear?
Babesiosis—because of its similar clinical presentation and ring
forms it can be misdiagnosed as malaria.
HELMINTHS
1. Cestodes
2. Nematodes
3. Trematodes
What nematode classically produces ova with mucous plugs at both ends
of the egg?
Trichuris trichiura (whipworm)
What are the two species included in the group commonly referred to as
hookworms?
1. Necator americanus
2. Ancylostoma duodenale
CLINICAL VIGNETTES
An HIV positive man presents with headache, neck pain and stiffness,
and confusion. A lumbar puncture is performed, what special tests
should be performed on the pa- tient’s CSF?
India ink staining to look for Cryptococcus (think capsule) and
cryptococcal anti-
gen
An 18-year-old girl presents with sore throat, low grade fever, and
fatigue. On exam- ination she is found to have nonexudative
pharyngitis, anterior and posterior cervical lymphadenopathy, and a
mildly enlarged spleen. What is the cause of her illness and what can
confirm the diagnosis?
EBV—confirm with Monospot (heterophile antibody test) and
peripheral blood smear may show atypical lymphocytes.
A 3-month-old infant is given honey as a cold remedy. The parents
bring the infant to the emergency room when the infant becomes quiet
and flaccid. What infection should be suspected and what is the
mechanism?
Clostridium botulinum—bacterial toxin inhibits release of
acetylcholine at neur- omuscular junctions
HEMATOLOGY
Anatomy/Histology
What cells are derived from pluripotent hematopoietic stem cells (think—
what are the components of a CBC with differential)?
What are the various types of leukocytes (white blood cells) and what are
their unique features?
See Table 5.1 on the following page.
Define anisocytosis:
The presence of an increased amount of RBC size variation
Define poikilocytosis:
The presence of an increased amount of RBC shape variation
Define reticulocytosis:
An increased number of immature RBCs
Pathology—RBCs
What labs would you order to differentiate iron deficiency anemia from
anemia of chronic disease?
See Table 5.2 below.
Figure 5.1 Schistocytes and bite cells among otherwise normal red
blood cells, a single polymorphonuclear cell, and platelets.
(Reproduced, with permission, from OHSU.)
How does the mutation in sickle cell anemia actually lead to anemia?
The mutation in hemoglobin leads to decreased red blood cell
deformability/elasti- city. As the RBCs traverse the capillaries they do
not have the normal elastic prop- erties to distort as they pass through,
instead the low oxygen tension promotes “sick- ling” of the cells. The
repeated “sickling” of the cells damages the cell membrane, this
damage in addition to the misshapen nature of the cells leads to
increased destruction of RBCs in the spleen.
What are some clinical findings associated with sickle cell anemia?
Anemia, cholelithiasis, pain crisis, dactylitis (painful and swollen
hands; and feet), and autosplenectomy
Which lab value measures the activity of the extrinsic pathway of the
coagulation cas- cade?
Prothrombin time (PT): from this measurement a standardized
number is calcu- lated and reported as the INR (international normalized
ratio).
1. Malignancy
2. Antiphospholipid antibodies (anti-cardiolipin antibodies and lupus
anticoagulants)
IMMUNOLOGY
Anatomy/Physiology
What is the reticuloendothelial system and what does it include?
It is part of the immune system, consisting of phagocytes,
primarily macrophages and monocytes, located in the reticular
connective tissue such as the spleen and lymphoid tissues. It includes
the bone marrow, thymus, spleen, lymph nodes, MALT (mucosa-
associated lymphoid tissues), Kupffer cells within the liver, and
microglia within the CNS.
Pathology—Immunodeficiencies
What are the four immunodeficiencies that affect B cells and how do
they differ in their mechanism?
1. Bruton agammaglobulinemia—X-linked recessive; decrease
production/number of B cells resulting in low levels of all
immunoglobulins; patients have multiple recur- rent bacterial
infections
2. Severe combined immunodeficiency (SCID)—most commonly X-
linked; de- creased production/number of B and T cells; patients
have multiple bacterial, viral, fungal, and protozoal infections
3. Wiskott-Aldrich syndrome—X-linked recessive; decreased
activation of B cells to encapsulated bacteria (↓ IgM); classic triad
of symptoms include pyogenic infec- tions, thrombocytopenic
purpura, and eczema
4. Selective IgA deficiency—unclear etiology, likely defect in isotype
switching; pa- tient have recurrent sinus and lung infections, may
also have milk allergies and diarrhea; anaphylaxis with transfusion
of blood products containing IgA
Pathology—Immune Responses
ONCOLOGY
What are the unique risks, benefits, and the chromosomal abnormality
associated with type M3 (promyelocytic) AML?
Increased risk of severe DIC, but responds well to treatment with
all-trans retinoic acid (vitamin A), including inducing differentiation
of myeloblast. Associated with t(15;17) translocation.
TRANSPLANTATION
What is an autograft?
Tissue implanted from self (donor and recipient are the same person)
What is an allograf t?
Tissue implanted from genetically different donor of the same
species as the recip-
ient
What is a xenograf t?
Tissue implanted from a donor of a different species (eg, porcine
heart valves in humans)
Describe GVHD:
The donor’s lymphocytes in the graft immunologically attack the
recipient’s cells (outside of the graft) because the graft lymphocytes
recognize the recipient’s cells as “non-self.”
TRANSFUSION MEDICINE
What does the direct antiglobulin test (DAT or direct Coombs) detect?
The DAT detects the presence of antibodies coating the patient’s
RBC surface in vivo.
How many units of whole blood does the average adult have in his/her
body?
8 to 10 units
What type of RBCs should be given to patients who have had previous
problems with febrile reactions to blood products, or who will be
chronically transfused?
Leukocyte-reduced RBCs since they decrease the risk of human
leukocyte antigen (HLA) autoimmunization and the risk of transfusion
reactions
What should be used as an intravascular volume expander?
Albumin, crystalloid, or colloid; not FFP
CLINICAL VIGNETTES
A woman is Rh∼ and pregnant with her second Rh+ child. If she does not
receive anti- Rh immunoglobulin during either pregnancy, what will
likely happen?
Erythroblastosis fetalis, or hemolytic disease of the newborn, will
result since the mother will likely have anti-Rh antibodies in her
bloodstream, from the first preg- nancy, that will cause hemolysis of
the second fetus’s RBCs.
A newborn baby shows signs of anemia and jaundice within the first 24
hours of life. What might he have?
Hemolytic disease of the newborn (due to Rh factor incompatibility
or ABO blood group incompatibility between mother and infant)
A patient develops an itchy, raised, red rash 2 to 3 days after using a new
brand of laundry detergent. What is this?
Contact dermatitis, which is a type IV hypersensitivity reaction
An alcoholic man presents with anemia. Would you expect his mean
corpuscular volume (MCV) to be high or low? What is he probably
deficient in?
Expect him to have megaloblastic anemia with a high MCV, either
as a direct res- ult from the chronic alcohol use or secondary to
concomitant vitamin B12 and/or folate deficiency.
A 53-year-old man has been feeling tired recently. His hemoglobin level
is 8.0. What should you be sure to order?
Check for occult blood in the stool and order a colonoscopy to
evaluate for pos- sible colorectal cancer.
An elderly woman has lung cancer. What might characterize her anemia
of chronic disease?
Decreased RBC life span, microcytosis, impaired iron metabolism,
and possible refractoriness or lack of response to erythropoietin
A 7-year-old girl has an elevated white blood cell (WBC) count, easy
bruising, and fa- tigue. What should she be evaluated for?
Acute lymphoblastic leukemia—the most common type of cancer in
children un- der age 15
A 17-year-old girl notices a swollen lymph node in her neck and biopsy
shows Reed- Sternberg cells. What disease is likely?
Hodgkin disease
CHAPTER 6
Cardiovascular Pathology
EMBRYOLOGY
The sinus venosus gives rise to which parts of the cardiovascular system?
A portion of the wall of the right atrium and the coronary sinus
The bulbus cordis gives rise to which parts of the cardiovascular system?
The proximal aorta and the pulmonary arteries
ANATOMY
Define the anatomic components of the cardiovascular system:
Heart, macrovasculature (aorta, arteries, large arterioles, veins),
microvasculature (small arterioles, postcapillary venules, capillaries), and
lymphatics
Name the four heart valves in the direction of blood flow:
1. Tricuspid valve
2. Pulmonary valve
3. Mitral valve
4. Aortic valve
What structure provides the base of attachment for the cardiac valves?
The so-called “fibrous skeleton” of the heart, which is composed
of dense con- nective tissue and has three main components: the septum
membranaceum, the trigona fibrosa, and the annuli fibrosi. The base of
each cardiac valve is attached to the annuli fibrosi.
What is the anatomic location of the carotid bodies and what is their
function?
The carotid bodies are located near the bifurcation of the common
carotid artery (bilaterally), and they function as chemoreceptors
monitoring levels of carbon dioxide and oxygen in the blood.
What is the anatomic location of the carotid sinus and what is their
function?
The carotid sinuses are dilatated segments of the internal carotid
artery (bilat- erally), and they contain baroreceptors which detect
changes in blood pressure and transmit this information to the central
nervous system.
Of what are the internal and external elastic lamina composed? What
are the func- tions of these structures?
Elastin. Gaps in the internal elastic lamina (aka fenestrae) allow
nutritive sub- stances from the blood to diffuse to cells located deeper
in the vessel wall. The elastic properties of both the internal and
external lamina serve to modulate the degree of pressure variation in
vessels during systole and diastole.
Which histologic layer of the wall of the heart is homologous with the
tunica intima of blood vessels?
Endocardium
What cells compose the myocardium and to what fibrous structure are
they associ- ated?
The myocardium is composed of cardiac muscle cells (aka
cardiomyocytes). These cells are arranged in layers which form a
complex spiral around the chambers of the heart. Many
cardiomyocytes are anchored to the fibrous cardiac skeleton.
PHYSIOLOGY
At which phase of the myocardial action potential does calcium enter the
cardiomyo- cytes?
Phase 2 (plateau)—calcium enters via voltage-gated calcium
channels
PATHOLOGY
General Principles
What is the leading cause of death in the United States?
Heart disease
What is hyperlipidemia?
A state of having elevated quantities of lipid substances—cholesterol
and trigly- cerides, in the blood
Vascular
What is atherosclerosis?
A process of thickening of the wall of any sized artery as a result of
deposition of fatty materials (eg, cholesterol) and subsequent chronic
inflammatory response
What are the risk factors associated with atherosclerosis?
Smoking; hypertension; hyperlipidemia; diabetes mellitus
Which drug relieves the chest pain associated with stable angina?
Usually nitroglycerin or vasodilators (ie, calcium channel
blockers). If there is de- creased oxygen-carrying capacity in the blood,
the patient may need blood transfusion or other therapies.
How is an MI diagnosed?
Clinical history and depending on timing of presentation, cardiac
enzymes, and abnormal ECG findings
1. Transmural
2. Subendocardial
What are the first cells to appear in the damaged tissue about 12 hours
post-MI?
Neutrophils
*Neutrophils go to a New site of injury at Noon (8-12 hours after
injury)
What is the most common cause of death within the first 24 to 48 hours
post-MI?
Arrhythmia
By the third day, what gross evidence is there that tissue has been
damaged by MI?
Area of infarct is pale and beginning to turn yellow with surrounding
hyperemia
What cells begin to migrate to the damaged tissue between days 3 and 5
post-MI?
Macrophages
What is the most common cause of death between days 3 and 7 post-MI?
Ventricular wall rupture leading to cardiac tamponade
If after MI, damaged myocardium does not regain full mobility, what
complication are patients at higher risk to experience?
Thromboembolic events due to blood stasis near the areas of
impaired myocardial mobility
What is eclampsia?
Seizures, plus the triad of preeclampsia
Inflammatory/Autoimmune
1. Acute pericarditis
2. Chronic pericarditis
What is cardiomyopathy?
Cardiomyopathy is a general term used to reflect a change in heart
muscle struc- ture or functional ability. There are many causes of
cardiomyopathy, including ac- quired and inherited forms, all of which
generally manifest clinically and pathologic- ally as one of the three
major types of cardiomyopathy.
Degenerative
What is the potential life-threatening event that can occur with AAA?
Rupture. Depending on the site of rupture, blood will fill the
retroperitoneum or abdominal cavity. The mortality of ruptured AAA is
estimated to be 75% to 90%.
What signs and symptoms would suggest that a patient is presenting with
tension pneumothorax?
Tachycardia, hypotension, decreased heart sounds, distended neck
veins, and ab- sent breath sounds on the side of the pneumothorax
Infectious
What are the small red lesions on palms and soles of patients with
bacterial endo- carditis?
Janeway lesions
What is the etiology of Osler nodes, Roth spots, and Janeway lesions?
All are caused in some way by immune complex deposition. In the
case of Roth spots and Janeway lesions, the immune complex
deposition occurs in vessels leading to small vessel vasculitis.
What is the usual time span between the tonsillitis infection and the onset
of rheum- atic fever?
1 to 4 weeks
Congenital
Which birth defect is associated with cyanosis, death soon after birth,
and maternal diabetes?
*
Transposition of the great vessels
Neoplastic
CLINICAL VIGNETTES
ANATOMY
HISTOLOGY
PHYSIOLOGY
What is surfactant?
Dipalmitoyl phosphatidylcholine—a complex lipoprotein that
coats the surface of alveoli, decreasing surface tension, and preventing
collapse at low lung volumes
What is compliance?
The change in lung volume produced by a given change in
intrapleural pressure (C = ΔV/ΔP)
Figure 7.1 Lung volumes.
What are the two types of respiratory sensors and where are they
located?
General Principles
What two conditions can result in alveolar pressure being greater than
arterial pres- sure?
Define shunting:
Deoxygenated blood passes through the pulmonary vasculature
without being ventilated.
What is dyspnea?
Shortness of breath
What is orthopnea?
Dyspnea occurring when the patient is in the supine position as a
result of a de- crease in vital capacity caused by abdominal contents
exerting force against the dia- phragm
What is atelectasis?
Alveolar collapse caused by bronchial obstruction or external
compression of the lung parenchyma by tumors, pleural fluid, or air
within the pleural cavity.
What are the Light criteria and how many must be met to diagnose an
exudative pleural effusion?
At least one of the following criteria must be met:
What are the most important factors for the survival of premature
infants?
Adequate vascularization and surfactant in the lungs; surfactant
generally begins to be produced at 32 weeks gestation.
Anatomic
What is a pneumothorax?
A collection of air or gas in the pleural cavity as a result of disease or
injury
What is a hemothorax?
A collection of whole blood in the pleural cavity caused by the
rupture of blood vessels resulting from trauma or inflammation
What is a chylothorax?
A pleural collection of a milky lymphatic fluid containing
microglobules of lipid
Inflammatory/Autoimmune
1. Asthma
2. Postnasal drip
3. Gastroesophageal reflux disease (GERD)
What are the two major categories of diffuse pulmonary lung disease?
1. Obstructive
2. Restrictive lung disease
What is asthma?
A condition characterized by episodic, reversible bronchospasm
resulting from an exaggerated bronchoconstrictor response to a variety
of stimuli
What is emphysema?
A condition characterized by the permanent enlargement of the
airspaces distal to the terminal bronchioles accompanied by destruction
of alveolar walls
1. Centrilobular
2. Panacinar
3. Paraseptal
What is α1-antitrypsin?
A glycoprotein which is a major inhibitor of serine protease
activity, particularly elastase, which is secreted by neutrophils during
inflammation
What is bronchiectasis?
The permanent dilation of bronchi and bronchioles due to the
destruction of muscle and elastic tissue secondary to infection or
obstruction caused by a variety of conditions
What is sarcoidosis?
A type IV hypersensitivity reaction to an unknown antigen that
results in a multisystem disease characterized by noncaseating
granulomas in multiple tissues and organs.
Environmental/Toxins
What is anthracosis?
An environmental disease caused by the inhalation of carbon dust.
It is usually en- demic in urban areas and causes no harm.
What is asbestosis?
An environmental disease caused by the inhalation of asbestos fibers
that result in diffuse pulmonary interstitial fibrosis
Vascular
Where do most pulmonary emboli arise?
More than 95% of PEs arise within the large deep veins of the lower
legs, typically the popliteal vein, femoral vein, and iliac vein.
Infection
What is pneumonia?
A respiratory disease characterized by inflammation of the lung
parenchyma (ex- cluding the bronchi) caused by viruses, bacteria, fungi,
or irritants
What are the four most common bacteria causing sinus and respiratory
infections?
1. Streptococcus pneumoniae
2. Haemophilus influenzae
3. Staphylococcus aureus
4. Mycoplasma pneumoniae
What is Q-fever?
The most common rickettsial pneumonia
What is ornithosis?
An atypical pneumonia that results from inhalation of the dried
excrement of birds infected with Chlamydia psittaci
What is tuberculosis?
A communicable, chronic granulomatous disease caused by
Mycobacterium tuberculosis
Define scrofula:
Tuberculous involvement of the oropharyngeal lymphoid tissue
with spread to the lymph nodes in the neck
1. Actinomyces israelii
2. Nocardia asteraids
What are the two types of inclusions seen in herpes simplex virus (HSV)
pneumonia?
What is a hamartoma?
A benign tumor-like nodule composed of an overgrowth of
otherwise normal ma- ture cells and tissues
CLINICAL VIGNETTES
A 7-year-old boy presents with his parents who note that he quickly
becomes short of breath when playing outside, especially in the spring
and fall. The boy will occasion- ally wheeze when he becomes short of
breath. With rest, his symptoms go away. What does this presentation
suggest?
Asthma, possibly triggered by exertion or allergens
A 35-year-old man complains of chest pain for the last 4 days. He states
that his pain is worsened by deep inspiration and describes it as a
sharp, stabbing pain. He tells you that he’s recovering from a recent
cold. What is the likely diagnosis?
Pleuritic chest pain secondary to viral infection
EMBRYOLOGY
Which adult structure does the umbilical vein ultimately give rise to?
The falciform ligament which contains the ligamentum teres
1. Septum transversum
2. Dorsal mesentery of the esophagus
3. Pleuroperitoneal folds
4. Body wall
What adult structures does the ventral pancreatic bud ultimately give rise
to?
Main pancreatic duct, uncinate process, and head of the pancreas
What adult structures does the dorsal pancreatic bud ultimately give rise
to?
Accessory pancreatic duct and body, isthmus, and tail of the pancreas
Into which three sections is the primitive gut divided?
ANATOMY
1. Falciform
2. Hepatoduodenal
3. Gastrohepatic ligaments
Name three major salivary glands associated with the oral cavity:
1. Parotid
2. Submandibular
3. Sublingual
HISTOLOGY
1. Epithelium
2. Lamina propria
3. Muscularis mucosa
What cell types are found within gastric glands and what do they
produce?
Congenital
Define atresia:
The absence of a luminal opening
Anatomic
What is achalasia?
The incomplete relaxation of the lower esophageal sphincter with
consequent dilatation of the proximal esophagus due to the loss of the
myenteric plexus
Trauma
What is leukoplakia?
An irregular, whitish patch/plaque caused by epidermal thickening
or hyperker- atosis and is believed to be a defense or reparative
response to insult. Whereas can- didiasis lesion can be scrapped off
with mechanical forces, leukoplakia cannot.
What are the most common sites of leukoplakia on the oral cavity?
The vermilion border of the lower lip, the buccal mucosa, and the
hard/soft palate
What is erythroplasia?
A clinical term used to describe lesions of the mucous membranes
which are red and nonulcerated
What are the most common locations of squamous cell carcinoma in the
oral cavity?
Ventral surface of the tongue, lower lips, floor of the mouth, gingiva,
and soft pal-
ate
What are the common risk factors for squamous cell carcinoma of the
esophagus?
• Alcohol consumption*
• Tobacco use*
• Long-standing esophagitis
• Achalasia
• Vitamins A, C, thiamine, or pyridoxine deficiency
• High content of nitrites/nitrosamine in diet
• Fungal contamination of food
• Familial
What is esophagitis?
Inflammation of esophageal mucosa
What is GERD?
Gastroesophageal reflux disease in which stomach acid refluxes into
the esophag-
us
STOMACH
Congenital
Infectious
What is H. pylori?
A spiral, microaerophilic, gram-negative bacterium that infects the
mucosal layer of the GI tract and produces urease and cytotoxins
Neoplasm
Inflammatory/Autoimmune
What clinical feature may help distinguish gastric ulcers from duodenal
ulcers?
Abdominal pain will be greater with meals if the patient has a
gastric ulcer and will decrease with meals if the patient has a duodenal
ulcer
Congenital
Anatomic
What is cholelithiasis?
The presence of stones (cholesterol, pigment, or mixed-type) in the
gallbladder
Infectious
What are the screening and confirmatory tests for chronic hepatitis B
infection?
• Screening—HBsAg in serum
• Confirmatory—hepatitis B virus, DNA, hepatitis B envelope
antigen (HBeAg), or hepatitis B core antigen (HBc Ag) in liver
What serum marker indicates hepatitis B immunity?
Anti-HBsAg (antihepatitis B surface antigen)
What are the screening and confirmatory tests for chronic hepatitis C
infection?
1. Hepatitis C
2. Hepatitis B
3. Hepatitis D
What viral hepatitis is associated with a high rate of acute liver failure in
pregnant women?
Hepatitis E
If you are suspicious that a patient has hepatitis A, what are the likely
laboratory find- ings?
Anti-HAV IgM (antihepatitis A virus immunoglobin M) and
increased levels of alanine transaminase (ALT), aspartate transaminase
(AST), bilirubin, and γ-glutamyl- transferase (GGT)
Environmental/Toxins
What is the most common cause of liver disease in the United States?
Alcohol abuse
Degenerative
What is cirrhosis?
End-stage liver disease with a loss of normal hepatic architecture
1. Alcohol abuse
2. Viral hepatitis (usually chronic hepatitis C)
What is the appearance of the cut surface of the liver in a patient with
chronic heart failure often referred to as?
Nutmeg liver, with areas of red centrilobular congestion alternating
with adjacent pale portal areas
What is kernicterus?
The accumulation of unconjugated bilirubin in the brain leading to
neurological damage
Inflammatory/Autoimmune
What is cholecystitis?
Acute inflammation of the gallbladder usually due to a gallstone
obstructing the cystic duct (cholelithiasis)
What is peritonitis?
Inflammation of the serosal lining of the abdominal cavity that
results from in- fection, injury, or associated with other diseases (eg,
spontaneous bacterial peritonitis associated with nephrotic syndrome)
Neoplasm
What is angiosarcoma?
A rare, malignant endothelial tumor of the liver
What is cholangiocarcinoma?
An adenocarcinoma of the biliary duct system
What is cholangiocarcinoma associated with?
It is associated with liver fluke (Opisthorchis sinensis) infestation,
primary scler- osing cholangitis, and thorotrast administration.
PANCREAS
Inflammatory/Autoimmune
• Trauma
• Hypertriglyceridemia
• Endoscopic retrograde cholangiopancreatography (ERCP)
• Medications—azathioprine, furosemide, glucocorticoids, and
cimetidine
• Infections—Ascaris lumbricoides, Opisthorchis sinensis, and
viruses (coxsackie and mumps)
• Hypercalcemia
• Scorpion stings (ie, Tityus trinitatis—rare!)
What are the signs and symptoms of acute pancreatitis?
Epigastric abdominal pain radiating to the back, nausea, vomiting,
and anorexia
What are the key lab findings in patients with acute pancreatitis?
Elevated amylase and lipase (higher specificity) with hypocalcemia
Neoplastic
What is the name of the sign associated with painless, palpable enlarged
gallbladder?
Courvoisier sign
What is the name of the sign associated with migratory thrombophlebitis
that often accompanies pancreatic or other adenocarcinoma?
Trousseau sign
INTESTINE
Congenital
What is an omphalocele?
An abdominal wall defect in which abdominal organs (intestine,
liver) are con- tained in a sac of peritoneum which protrudes through the
umbilicus
What is gastroschisis?
An abdominal wall defect in which abdominal organs are present
outside of the fetal body but are not contained within a sac of
peritoneum
Anatomic
What are the major causes of intestinal obstruction?
Hernias, adhesions, intussusception, paralytic ileus, volvulus,
tumors, obstructive gallstones, bowel infarction, foreign bodies,
congenital bands, meconium, imperforate anus, myopathies, and
neuropathies
What is intussusception?
The telescoping of a proximal segment of the bowel into the
immediate distal seg- ment
What is volvulus?
Twisting of a loop of bowel or other structure about its base of
attachment
Vascular
Infectious
Name two bacteria which can infect the intestine that are nonlactose
fermenters:
1. Salmonella
2. Shigella
1. Rotavirus
2. Norovirus
3. Adenovirus
What is Whipple disease and what are the associated signs and
symptoms?
A systemic infection—often associated with malabsorption,
abdominal pain, arth- ralgias, intestinal lipodystrophy, and diarrhea
What are the classic histologic features associated with Whipple disease?
Foamy macrophages with PAS-positive inclusions infiltrating the
lamina propria of the colon
Inflammatory/Autoimmune
What is a diverticulum?
An outpouching from a fluid-filled or hollow organ (eg, the bladder,
esophagus)
What is the condition called when a patient has diverticula in their
colon?
Diverticulosis
What is diverticulitis?
Inflammation of one or more diverticulum
What are the most common benign tumors of the small intestine?
Polyps, gastrointestinal stromal tumors (GIST), adenomas, and
lipomas
What are the most common malignant tumors of the small intestine?
Adenocarcinomas and carcinoid tumors
The majority of cancers in the large intestine are what kind of cancers?
Adenocarcinomas
CLINICAL VIGNETTES
EMBRYOLOGY
What three sets of nephric structures does the nephrogenic cord develop
into?
1. Pronephros
2. Mesonephros
3. Metanephros
1. Mesonephric tubules
2. Mesonephric duct (Wolffian duct)
What nephric structure develops into the definitive adult kidney?
The metanephros
The kidneys ascend from the sacral region in fetal life to vertebral
levels T12-L3 in the adult. This upward migration is accompanied by
what other event?
The medial rotation of the kidneys by 90°
What are the three parts of the male urethra that are derived from
endoderm?
Endodermal outgrowths from what part of the male urethra develop into
the bulbourethral glands (Cowper glands)?
Membranous urethra
The distal part of the penile urethra is derived from what ectodermal
structure?
The glandular plate
The glandular plate joins the penile urethra and becomes canalized to
form what structure?
The navicular fossa
HISTOLOGY
The metanephric vesicles differentiate into various structures that
together form a nephron. What are the structures?
1. Glomerulus
2. Bowman capsule
3. Proximal convoluted tubule
4. Loop of Henle
5. Distal convoluted tubule
6. Connecting duct
1. Glomeruli
2. Tubules
3. Interstitium
4. Vessels
What type of tissue lines the ureter, pelvis, major calyx, and minor
calyx?
Transitional epithelium
RENAL PATHOPHYSIOLOGY
General Principles
What is renal failure (end-stage nephropathy)?
The inability of the kidneys to excrete wastes and maintain
electrolyte balance. It can be broadly divided into acute and chronic
renal failure. See Table 9.1 for mani- festations of electrolyte
abnormalities.
1. Hyperkalemia
2. Central fluid overload
3. Metabolic acidosis
4. Severe hyperphosphatemia
5. Severe uremia
Congenital
What is renal agenesis?
The failure of development of one (unilateral) or both (bilateral)
kidneys
What effects does bilateral renal agenesis (BRA) have on the developing
fetus?
Fetal kidneys are necessary for amniotic fluid production. As such,
there will be less than the normal amount of amniotic fluid present
(oligohydramnios) which will subsequently impair pulmonary
development.
Inflammatory/Autoimmune
Glomerular disease
What are the primary glomerular diseases that can cause nephrotic
syndrome?
Minimal change disease (lipoid nephrosis); focal segmental
glomerulosclerosis; membranous glomerulonephritis
What are the known disorders or agents that are associated with
membranous glom- erulonephritis?
How does the World Health Organization (WHO) classify the pattern of
renal in- volvement in SLE?
• Type I—no renal involvement
• Type II—mesangial form characterized by focal and segmental
glomerular in- volvement with an increase in both the number of
mesangial cells and mesangial matrix
• Type III—focal proliferative form
• Type IV—diffuse proliferative form with glomerular changes that
result in scarring and renal failure. **This is the most common and
most severe subtype.
• Type V—membranous form that is indistinguishable from primary
membranous glomerulonephritis
What are the primary glomerular diseases and systemic diseases that
cause nephritic syndrome?
Immunoglobin A (IgA) nephropathy or Berger disease; acute
poststreptococcal glomerulonephritis; rapidly progressive (crescentic)
glomerulonephritis; membrano- proliferative glomerulonephritis;
Goodpasture disease; Wegener granulomatosis; Al- port syndrome;
microscopic polyangiitis
1. Goodpasture syndrome
2. Wegener granulomatosis
Tubulo-interstitial disease
What is cystinuria?
An autosomal recessive disorder that causes impaired renal tubular
reabsorption of cystine, ornithine, lysine, and arginine
What is nephrocalcinosis?
Diffuse deposition of calcium in the kidney parenchyma that can
lead to renal fail- ure. It is often caused by hypercalcemia and
hyperphosphatemia.
Collecting system
Drugs
Anatomic
What is obstructive uropathy?
Obstruction occurring anywhere along the urinary tract from the
kidneys to the ur- ethral meatus
What is urolithiasis?
A condition in which crystals combine to form stones in the urinary
tract
What is hydronephrosis?
Dilation of the renal pelvis, calices, and sometimes, the collecting
ducts secondary to obstruction of urine flow by calculi, tumors,
neurologic disorders, or congenital an- omalies
1. Calcium
2. Magnesium ammonium phosphate
3. Uric acid
4. Cystine
What are the key characteristics of calcium stones?
Most common type of kidney stone (80%-85%); consist of calcium
oxalate, calci- um phosphate, or both; are radiolucent; tend to recur
What disorders or conditions cause hypercalcemia and result in
hypercalciuria with renal stone formation?
Cancers, increased parathyroid hormone (PTH), increased vitamin
D, milk-alkali syndrome, and sarcoidosis—all lead to hypercalcemia
and subsequent stone formation
How does the solubility of uric acid affect treatment of uric acid stones?
The solubility of uric acid depends on the acidity or alkalinity of
the urine. In acid urine, uric acid crystals precipitate leading to stone
formation. In alkaline urine, uric acid remains soluble. Treatment,
therefore, involves alkalinization of urine.
Why are most infections of the urinary tract and kidney seen in females?
Due to the shorter length of the female urethra
What are the common predisposing factors that increase the risk of
urinary tract in- fection?
Obstruction of urinary flow; surgery on the kidney or urinary tract;
catheters in- serted through the urethra into the bladder; gynecologic
abnormalities; diabetes; preg- nancy
What are the major clinical and laboratory findings associated with
acute pyeloneph- ritis?
What are the additional clinical and laboratory findings associated with
acute pyel- onephritis?
Fever, leukocytosis, flank tenderness, urinary white cells, and white
cell casts in the urine
Neoplastic
CLINICAL VIGNETTES
EMBRYOLOGY
What are the two types of genital ducts found in the undifferentiated
embryo?
The urogenital systems of both males and females develop from what
embryologic structure?
Urogenital ridge
What is the name of the thickening along the urogenital ridge from
which the gonads develop?
Gonadal ridge
What develops from the gonadal ridge and contains the primordial germ
cells?
Primary sex cords
The gonads initially develop in the abdomen and then descend into the
pelvis in fe- males or into the scrotum in males. Which embryologic
structure is involved in this descent?
Gubernaculum
In the female, the mesonephric ducts and tubules become part of the
urinary system and eventually regress after the formation of what
structure?
Metanephric kidneys
In males, the mesonephric (wolffian) duct and tubules develop into what
structures?
Seminal vesicles, epididymis, ejaculatory duct, and ductus deferens
*Mesonephric ducts and tubules SEED
Some mesonephric tubules in the testes develop into what structures?
The efferent ductules of the testes
ANATOMY
What two events must take place after ejaculation before a sperm can be
fully func- tional and capable of fertilization?
1. Capacitation
2. Acrosome reaction
What is capacitation?
A series of enzymatic and biochemical events which occur while
sperm are in the female genital tract prior to fertilization. The result of
capacitation is that sperm are hypermotile and have destabilized
plasma membranes facilitating initiation of the ac- rosome reaction.
The first meiotic division is completed before ovulation takes place. The
cell then con- tinues onto a second division that is also arrested. At
what stage of meiosis does this second arrest occur?
Metaphase
During meiosis, a primary oocyte will divide into four daughter cells.
Of the four daughter cells, only one will become an ovum and the rest
will degenerate. What are the cells that degenerate called?
Polar bodies
PATHOLOGY
Congenital
What is a hypospadia?
A congenital abnormality in males that results in the displacement
of the urethral meatus
What is epispadias?
A congenital abnormality more commonly seen in males in which
the urethra is on the dorsal surface of the penis (or results in a bifid
clitoris in females). Of note, epispadias is not a type of hypospadias.
What is a chordee?
A congenital malformation of unknown etiology that results in the
downward dis- placement of the penis
What is polythelia?
Also known as supernumerary nipple, polythelia is development of
a nipple along the “milk line” which extends from bilateral axilla to the
groin.
What is polymastia?
Also known as supernumerary breast, polymastia is development
of glandular breast tissue with or without an associated nipple and also
usually along the “milk line.”
Inflammation
What is balanitis?
A nonspecific inflammation of the glans penis and prepuce that is
caused by phys- ical trauma, irritation, or infection
Neoplastic
What is the most common gynecologic malignancy in the United States?
Endometrial carcinoma
HPV DNA sequences are often integrated into the genome of dysplastic
or malignant cervical epithelial cells. What is the molecular mechanism
associated with this pro- cess?
HPV viral proteins E6 and E7 bind and inactivate the gene
products of p53 and
Rb, both tumor suppressor genes, thus allowing the cells to accumulate
DNA damage.
What are two most common tumors that arise in the fallopian tubes?
What is a teratoma?
A tumor that exhibits evidence of simultaneous differentiation from
all three germ layers—endodermal, mesodermal, and ectodermal lines.
These elements may be ma- ture or immature.
What is a fibroma?
A solid tumor consisting of bundles of spindle-shaped fibroblasts
Where are the primary tumors that metastasize to the ovaries usually
located?
Gastrointestinal tract, breast, and endometrium
What is a fibroadenoma?
A benign breast tumor of the intralobular stroma that presents as a
firm, rubbery, painless, well-circumscribed mass
What are the general treatments available to women with breast cancer?
Surgery (breast-conserving vs mastectomy), radiotherapy,
chemotherapy, hor- mone therapy, and pain management
What are the important risk factors associated with testicular tumors?
1. Cryptorchidism
2. Genetic factors
3. Testicular dysgenesis
What is a seminoma?
The most frequently occurring germ cell tumor that presents as
painless enlarge- ment of the testis
What tumor marker can be found in the serum of patients with testicular
seminoma?
hCG
A patient newly diagnosed with seminoma asks you about treatment and
prognosis. What do you tell him?
Though malignant, seminomas are very radiosensitive and can often
be cured.
What tumor marker can be found in the serum of patients with yolk sac
tumors?
AFP
What is a testicular choriocarcinoma?
A highly malignant neoplasm composed of both cytotrophoblastic
and syncytio- trophoblastic elements that is often encountered as a
component of mixed germ cell tumors
hCG
Mixed germ cell tumors have variable prognosis. What feature usually
dictates pro- gnosis?
The least mature element making up the mixed germ cell tumor
What are the available treatments for most germ cell tumors?
Treatment usually includes radiation and chemotherapy, depending
on the histolo- gic type of the neoplasm. In particular, chemotherapy
has dramatically improved the prognosis of nonseminomatous germ
cell tumors.
Infectious
What is candidiasis?
The most common form of vaginitis, caused by Candida albicans
What conditions are frequently associated with candidiasis?
Diabetes mellitus, pregnancy, broad-spectrum antibiotic therapy, oral
contracept- ive use, and immunosuppression
What is trichomoniasis?
A sexually transmitted type of vaginitis caused by Trichomonas
vaginalis
What vaginal infection is known for its fishy odor and can be treated
with met- ronidazole?
Bacterial vaginosis (Gardnerella vaginalis)
Which of the herpes simplex viruses is associated with genital herpes and
spreads via sexual contact?
Herpes simplex virus (HSV) type 2
What is chancroid?
A sexually transmitted disease caused by Haemophilus ducreyi that
is character- ized by painful, ulcerated lesions
Define cervicitis:
An inflammation of the cervix caused by a number of different
organisms includ- ing staphylococci, enterococci, Gardnerella
vaginalis, Candida albicans, Trichomo- nas vaginalis, Chlamydia
trachomatis, and Neisseria gonorrhoeae
What is salpingitis?
Inflammation of the fallopian tubes secondary to infection, trauma,
or surgical ma- nipulation
What is orchitis?
Swelling/inflammation of the testes secondary to viral or bacterial
infection. When viral, it is most often due to mumps; when bacterial, it
is often associated with epidi- dymitis.
What is epididymitis?
An inflammation or infection of the epididymis due to the
retrograde extension of organisms from the vas deferens
Organisms that cause epididymitis will vary with the age of the patient.
List the or- ganisms and the typical age groups they affect.
Anatomic
What is endometriosis?
A nonneoplastic condition caused by the ectopic dissemination of
endometrial tis- sue to the ovaries or other structures outside the uterus
What is adenomyosis?
A condition characterized by the extension or presence of ectopic
endometrial tis- sue in the myometrium
A patient with a corpus luteum cyst would likely present with what sign
or symptom?
Menstrual irregularity
What is priapism?
A persistent, abnormal, and painful erection of the penis that
develops when blood becomes trapped and is unable to drain
What is cryptorchidism?
Failure of one or both of the testicles to descend into the scrotum
What is a hydrocele?
A painless swelling of the scrotum caused by a collection of fluid
around the testicle which results from a small patency in the processus
vaginalis
What is a chylocele?
An accumulation of lymphatic fluid within the tunica vaginalis
secondary to lymphatic obstruction
What is a hematocele?
The abnormal accumulation of blood distending the tunica vaginalis
of the testis, often secondary to trauma or tumor
What is a varicocele?
A dilatation of the veins associated with the spermatic cord in the
testes
What is a spermatocele?
A cyst of the epididymis containing sperm
Obstetrical
What is a placental abruption (abruptio placentae)?
The premature detachment of the placenta from the wall of the
uterus causing severe antepartum bleeding and potentially fetal death
What will you see after delivery of a baby in a patient with placenta
accreta?
Massive hemorrhage
What is choriocarcinoma?
A highly malignant neoplasm that arises from the cells in the
chorion layer of the placenta. It is typically composed of both
cytotrophoblastic and syncytiotrophoblastic elements.
Intersex Conditions
What must be done with testes that are found in the labia majora of a
patient with complete androgen insensitivity?
They must be removed to circumvent malignant tumor formation.
CLINICAL VIGNETTES
PITUITARY
General Principles
What is the visual field defect that occurs in patients with pituitary
adenomas?
Bitemporal hemianopsia
What is acromegaly?
The result of continued stimulation by excess growth hormone
(GH) after closure of the epiphyseal plates (ie, adults), characterized
by frontal bossing (prominent fore- head), large head, nose, hands,
protruding jaw, thick tongue, and deepening of the voice
What will serum and urine lab tests find in patients with central DI?
What is the dreaded complication that may occur with rapid correction
of sodium levels in a patient with SIADH?
Central pontine myelinolysis—acute, noninflammatory
demyelination of neurons occurring predominately within the pons of
the brain stem
THYROID
General Principles
Hyperthyroidism
What are the symptoms of hyperthyroidism?
Palpitations, weakness, nervousness/anxiety, weight loss, diarrhea,
intolerance to heat, tremor
What other diseases are commonly found in people with Grave disease?
Systemic lupus erythematosus (SLE); pernicious anemia; diabetes
mellitus (DM) type I; Addison disease
Hypothyroidism
For what type of cancers are people with Hashimoto thyroiditis at higher
risk?
B-cell lymphomas of the thyroid gland
Neoplastic
A young adult, female patient has a solitary, painless neck mass. What
is the most likely diagnosis?
Thyroid adenoma
What are some features that make a lesion of the thyroid suspicious for
cancer?
Solitary lesion; radiation history; cold nodule; female sex
What are the three most important things to remember about medullary
carcinoma?
General Principles
Hyperparathyroidism
What are the clinical symptoms of primary hyperparathyroidism?
Fatigue; hypercalcemia symptoms—“stones, bones, groans, and
psychiatric over- tones”
Hypoparathyroidism
What is tetany?
Neuromuscular spasm/irritability
ADRENAL GLANDS
Embryology
General Principles
1. Adrenal cortex
2. Adrenal medulla
1. Zona g lomerulosa
2. Zona f asciculate
3. Zona r eticularis
*“G, F, R”
Hypercortisolism
What are the classic clinical findings associated with Cushing syndrome?
Weight gain, hypertension, truncal obesity, moon facies, abdominal
striae, and ac- cumulation of fat on the posterior neck
What is the technical term for the accumulation of fat on the posterior
neck in Cush- ing syndrome?
Buffalo hump
What is the most common cause of Cushing syndrome?
Exogenous steroid administration (drugs)
What lab values suggest that Cushing syndrome is due to ectopic ACTH
from a non- endocrine neoplastic origin?
Increased ACTH; no suppression of ACTH with any level of
dexamethasone
Hyperaldosteronism
What are the clinical findings, including lab values, associated with
hyperaldosteron- ism?
Hypertension, water retention (weight gain ± edema), muscle
wasting, paresthesi- as; labs—hypokalemia, hypernatremia, and
metabolic alkalosis
Adrenal Insufficiency
What HLA types are associated with the autoimmune form of Addison
disease?
HLA-B8 and HLA-DR3
Neoplastic
What are the two most common tumors of the adrenal medulla?
Pheochromocytoma and neuroblastoma
What is a pheochromocytoma?
A tumor of chromaffin cells of the adrenal medulla
What is a neuroblastoma?
Malignant neuroendocrine tumor of childhood, arising from neural
crest cells, that most commonly originates in adrenal medulla (∼50%)
but can arise from any neural tissue; most common solid tumor in
infancy (most common extracranial solid tumor of childhood)
PANCREAS
General Principles
Diabetes Mellitus
What are the common presenting symptoms of diabetes mellitus (DM)?
Polydipsia, polyphagia, polyuria, weight loss
1. β-Hydroxybutyric acid
2. Acetoacetic acid
3. Acetone
What is the rapid and deep breathing found in DM type I known as?
Kussmaul breathing
What is the first-line treatment in DM type I?
Insulin and hydration
What is the treatment of choice if diet and exercise do not lower fasting
glucose levels?
Oral hypoglycemic agents
What organ systems are especially at risk with DM types I and II?
Cardiovascular; kidney; eye (retina); nervous system
What are patients with DM at higher risk for which affects the blood
vessels and coronary arteries?
Atherosclerosis
NEOPLASMS
• Medulla (adrenal)—pheochromocytoma
• Medullary carcinoma of thyroid
• Parathyroid (hyper)
*MEN2A has problems with a pair of medullas and parathyroids!
What glands/organs are affected by MEN3 (MEN2B)?
CLINICAL VIGNETTES
A 42-year-old woman presents with increasing nausea, vomiting, and
headache for the past month. On physical examination, she has
abnormal vision in the temporal fields. What is the most likely
diagnosis?
Pituitary adenoma
A 28-year-old woman presents with amenorrhea, galactorrhea, nausea,
vomiting, and fatigue. What common laboratory test should be
included in the initial diagnostic work-up?
Beta-human chorionic gonadotropin (β-HCG)/urine pregnancy test
A 35-year-old female presents with fatigue and pain in the neck, jaw,
and throat. She has symptoms of hypothyroidism and had symptoms of
hyperthyroidism 1 week ago. She reports that she just recovered from
an upper respiratory tract infection (URI). What is the most likely
diagnosis?
de Quervain thyroiditis
A term female infant is born with ambiguous genitalia and low blood
pressure. Labs show increased serum potassium and hyponatremia.
What is the most likely diagnos- is?
Congenital adrenal hyperplasia (CAH)
EMBRYOLOGY
What structures or cells related to the nervous system are derived from
neuroecto- derm?
The pineal gland, neurons of the central nervous system,
oligodendrocytes, and as- trocytes
What structures or cells related to the nervous system are derived from
neural crest?
Schwann cells, dorsal root ganglia, autonomic ganglia, and pia mater
ANATOMY
What are the anatomic components of the central nervous system (CNS)?
The brain and the spinal cord
Name the three major spinal tracts and describe the type of transmitted
information:
HISTOLOGY
What cells form myelin sheets around axons in the peripheral nervous
system?
Schwann cells
NEUROPATHOLOGY
General Principles
Is the normal range of ICP low or high relative to mean arterial pressure
(MAP)?
Low. Increases in ICP (eg, by mass lesions, increased amount of
CSF, or bleeding) can quickly cause neurologic compromise.
What type of edema results from the influx of sodium and water in the
neural cells?
Cytotoxic edema—which is due to inadequate function of the
sodium-potassium pump in glial cells. The blood-brain barrier remains
intact.
What is hydrocephalus?
A condition in which there is increased fluid within the skull
Congenital
What is microcephaly?
Head circumference smaller than two standard deviations below
the mean for age and sex; there are numerous etiologies for
microcephaly including infectious and ge- netic causes and maternal
alcohol use.
What is macrocephaly?
Head circumference larger than two standard deviations above the
mean for age and sex; there are numerous etiologies for macrocephaly
including hydrocephalus and genetic, infectious, and environmental
causes.
What is anencephaly?
Absence of a large part of the brain and skull; results when the
cephalic end of the neural tube fails to close (around day 23-26 of
gestation)
What are the typical clinical findings of Klüver-Bucy syndrome?
Hypersexuality, uninhibited behavior, visual agnosia, and
hyperorality
What visual field defect will a patient experience if a lesion involving the
right optic nerve is present?
Right anopia (blindness of the right temporal and right nasal visual
fields)
What visual field defect will a patient experience if a lesion involving the
optic chiasm is present?
Bitemporal hemianopia (blindness of the left temporal and right
temporal visual fields)
What visual field defect will a patient experience if a lesion involving the
right optic tract is present?
Left homonymous hemianopsia (blindness of the left temporal and
right nasal visual fields)
What visual field defect will a patient experience if a lesion involving the
dorsal optic radiation is present?
Left lower quadrantanopsia
What is a seizure?
A transient condition of excessive or synchronous neuronal activity
in the brain
What is epilepsy?
A disorder in which a patient experiences recurrent seizures (does
not include feb- rile seizures)
What is the difference between a partial and a generalized seizure?
Partial seizures affect only one, localized part of the brain,
therefore the clinical symptoms will be specific to the area of affected
brain. Generalized seizures affect the brain diffusely, therefore the
clinical symptoms will be generalized and nonlocalizing.
What is the difference between a simple and a complex seizure?
Simple and complex seizures are subtypes of partial seizures. A
complex partial seizure is one that starts localized (as a simple partial
seizure) but then secondarily generalizes thereby impairing
consciousness. There is no alteration of consciousness in a simple
seizure.
Neoplastic
What are the common presenting features of patients with brain tumors?
Patients may present with a variety of symptoms including nausea,
headache, seizures, focal findings (eg, compression of a single cranial
nerve), and/or altered mental status/confusion.
What is the most common adult brain tumor?
Metastases (eg, lung, breast, melanoma)
Within the skull, where are adult brain tumors most often located?
Superior to the tentorium—“supratentorial”
Within the skull, where are pediatric brain tumors most often located?
Inferior to the tentorium—”infratentorial”
What is a “glioma”?
A glioma is a relatively nonspecific term applied to any brain tumor
derived from glial cells which include astrocytes, oligodendrocytes, and
microglia.
Vascular
Inflammatory/Autoimmune
What demyelinating disease occurs after viral infection and, unlike MS,
is self-lim- ited?
Acute disseminated encephalomyelitis
Infectious
What is meningitis?
An inflammatory process of the leptomeninges and cerebrospinal
fluid (CSF) loc- ated within the subarachnoid space, usually associated
with an infectious organism
1. Meningovascular neurosyphilis
2. Paretic neurosyphilis
3. Tabes dorsalis
1. Cryptococcus
2. Candida meningitis; many others are possible
Degenerative
What are the two most common causes of dementia in elderly patients?
1. Alzheimer disease
2. Multi-infarct (vascular) dementia
Traumatic
When will a person with hemisection of the spinal cord present with
features of Horner syndrome?
When the hemisection occurs above the level of T1
Environmental/Toxins
CLINICAL VIGNETTES
A 60-year-old woman can speak, but her words make no sense. Where
is the brain le- sion?
Wernicke area
A 45-year-old woman has had multiple car wrecks because she states she
has lost her peripheral vision. Where is the lesion?
Optic chiasm
A 20-year-old man is hit in the side of the head with a baseball. He has
a 30-minute lucid interval followed by headache and decreased level of
consciousness. What is his diagnosis?
Epidural hematoma
A 90-year-old man trips and falls. A few days later, he develops mental
status changes and contralateral hemiparesis. What is his diagnosis?
Subdural hematoma
A 10-year-old boy has sudden spells where he falls to the ground with
complete loss of muscle tone. What is the diagnosis?
Atonic seizure
A 70-year-old woman presents with a badly burned leg and says she
has decreased pain and temperature feeling on the right side of her
body from her breast down. On examination, her proprioception and
discriminative touch are normal. What somato- sensory pathway is
affected?
Spinothalamic pathway
A 31-year-old man has right-sided proprioception and discriminative
touch sensory loss, and left-sided pain and temperature sensory loss
from hemisection of his sixth thoracic vertebrae. What is the syndrome
called?
Brown-Séquard syndrome
EMBRYOLOGY
ANATOMY/HISTOLOGY
1. Epidermis
2. Dermis
3. Subcutaneous tissue
1. Papillary layer
2. Reticular layer
What is the name of tactile disks that mediate light crude touch?
Merkel corpuscles
PATHOLOGY
General Principles
What is a macule?
Flat, discolored (hypo- or hyper-pigmented) area of skin <1 cm in
diameter
What is a patch?
Flat, discolored area of skin >1 cm in diameter
What is a papule?
Raised area of skin of any color that is <1 cm in diameter
What is a plaque?
Raised area of skin of any color that is >1 cm in diameter
What is a nodule?
A palpable, roughly round lesion arising in the dermis or
subcutaneous tissues
What is a vesicle?
A raised, fluid-filled blister measuring <0.5 cm in diameter
What is a bulla?
A raised, fluid-filled blister measuring >0.5 cm in diameter
What is a pustule?
A blister that is filled with pus (generally bacteria and necrotic
debris)
What is a wheal?
A “hive,” generally a round lesion resulting from edema in the
dermis
What is an erosion?
The skin lesion that results when all or part of the epidermis is
removed (ie, ab- raded), will not leave a scar
What is an ulcer?
Full-thickness loss of epidermis and loss of all or part of the
epidermis, will leave a scar
Define hyperkeratosis:
Excessive keratin production leading to thickening of the stratum
corneum
Define lichenification:
Visual appearance of thickened skin with prominent skin markings
that occurs sec- ondary to chronic scratching of itchy skin, can be
associated with atopic dermatitis
Define ichthyosis:
Excessive cornification of the skin, giving it a scaly appearance
Define hypertrichosis:
Excessive hairiness due to increased formation of hair follicles,
may be regional or generalized
Congenital/Inherited
What is a hemangioma?
The most common tumor of infancy, hemangiomas are benign
vascular prolifera- tions which can appear in the skin (most often on the
face or scalp). Flat, larger lesions may be referred to as port-wine
stains.
Which rash often described as a target lesion that has a red center, pale
zone, and a dark outer ring (targetoid)?
Erythema multiforme
Which disease results from the deposition of collagen in skin that causes a
“hardened” and “thickened” appearance and is associated with Raynaud
phenomenon?
Scleroderma
What is scleroderma?
Also known as systemic sclerosis, scleroderma is a chronic disease
characterized by accumulation of fibrous tissue in the skin and other
organs. The etiology is un- known.
What is vitiligo?
Partial or complete loss of melanocytes within the epidermis
Infectious
Viral Exanthems
Describe the rash associated with herpes simplex type I:
Small recurrent painful vesicles involving oral mucosa; recurrent
events may ap- pear to be related to stressful life events or periods of
other illness.
• HPV-1—planter/palmer warts
• HPV-2—common warts, some forms of plantar warts
• HPV-3—flat warts
What is the rapid developing infection of the skin and fascia that may
lead to death if not treated quickly?
Necrotizing fasciitis
What are the organisms responsible for necrotizing fasciitis?
Group A streptococci or Clostridium perfringens
What is used to test for typhus and Rocky Mountain spotted fever?
A positive Weil-Felix reaction—tests for cross-reaction of
antirickettsial antibod- ies with Proteus antigen
What is the plague native to the United States that lives in squirrels and
prairie dogs?
Sylvatic plague
What is the easiest and quickest way to determine if the etiology of a skin
rash is a fungus?
KOH preparation—will see fungal forms on microscopy
Actinic keratosis lesions may transform into what type of skin cancer if
left un- treated?
Squamous cell carcinoma
What are some risk factors for squamous cell carcinoma?
Sun exposure; ionizing radiation; actinic keratosis;
immunosuppression; arsenic; industrial carcinogens
What are tan/brown plaques or papules that have a “stuck on”
appearance and may be found anywhere on the body of adults, except
the palms and soles?
Seborrheic keratosis
Large (>20 cm) congenital nevi and dysplastic nevi may be precursor
lesions for what type of cancer?
Melanoma
Traumatic/Degenerative
What is the most common type of collagen in a keloid?
Type III collagen
What are the risk factors for keloid formation?
African American race, <30 years of age, and increased skin tension
in a wound
Which kind of burn usually blisters and affects the dermis and adnexal
structures?
Second-degree burn
Which type of burn involves the entire thickness of the skin, including
variable amount of underlying fat and causes loss of sensation in
affected areas?
Third-degree burn
Drugs/Toxins
Which drug causes red man syndrome usually during rapid intravenous
infusion?
Vancomycin
Metabolism
What is the most likely vitamin deficiency that manifests as petechiae,
ecchymoses, abnormal hair growth, bleeding gums, and poor wound
healing?
Vitamin C (scurvy)
CLINICAL VIGNETTES
A mother brings her 5-year-old son to the physician because she noted
her son scratching a pinkish lesion on his neck. Upon examination, the
physician notes a ring- shaped scaling plaque with central clearing and
elevated borders. What is the most likely diagnosis?
Tinea corporis
A 40-year-old man has a rash of scaly red patches on his trunk, face,
and extremities. A biopsy is taken and superficial dermal infiltrates of
T lymphocytes and a collection of atypical lymphocytes are seen within
the epidermis. What is the most likely dia- gnosis?
Cutaneous T-cell lymphoma (Mycosis fungoides)
EMBRYOLOGY
From what embryologic tissue type are muscle, bone, and connective
tissue derived?
Mesoderm
What are the two embryologic processes by which bone may be formed?
ANATOMY
What are the three general types of bones in the human body?
1. Long bones
2. Flat bones
3. Short, tubular bones
What are the four tendons that comprise the rotator cuff?
1. Supraspinatus
2. Subscapularis
3. Infraspinatus
4. Teres minor
What is osteoid?
A protein material produced by osteoblasts which will mineralize to
become bone
What cell type is responsible for creating calcified columns into which
osteoblasts will migrate?
Chondrocytes
What is found in the medullary cavity of long bones?
Trabecular bone, bone marrow, blood vessels
What is a sarcomere?
The smallest contractile unit in skeletal muscle. Regions of the
sarcomere that are visible histologically, include I-band, A-band, H-
band, Z-line, and M-line.
PATHOLOGY
Congenital
Anatomic
What is osteopetrosis?
A condition resulting from the failure of normal bone resorption
which results in thickened, dense bones
Traumatic
Degenerative
What is osteoporosis?
Most often, an age-related reduction in bone density and mass
What are the two dominate clinical patterns of osteoporosis?
1. Postmenopausal
2. Senile
Name two fractures that are common among osteoporotic patients:
Inflammatory/Autoimmune
What is podagra?
Gout of the metatarsophalangeal (MTP) joint of the big toe
What is gout?
A condition in which monosodium urate crystals precipitate in joint
spaces due to hyperuricemia
How are the crystals of pseudogout different from the crystals of gout?
Calcium pyrophosphate (pseudogout) instead of monosodium urate
(gout) and weakly positively birefringent (pseudogout) instead of
negatively birefringent (gout)
Which lab test should you order when you suspect RA?
Rheumatoid factor (RF)
What disease would be expected in a young woman that suffers with RA-
like symp- toms (polyarthritis), leukopenia, leg ulcers, and
splenomegaly?
Felty syndrome
1. Ankylosing spondylitis
2. Psoriatic arthritis
What is the name of the sign that occurs when a small amount of scale
is removed from a psoriatic plaque, leaving small bleeding points
behind?
Auspitz sign
What cardiac lesion is associated with SLE in the adult and consists of
nonbacterial verrucous valvular vegetations?
Libman-Sacks endocarditis
What are some other causes for chest pain in a patient with SLE?
Pleuritis; pericarditis
What are wire-loop lesions in the kidney and what do they represent?
Thickening of the capillary wall found in diffuse proliferative
glomerulonephritis (GN); indicate a poor prognosis with SLE
1. Anti-Smith antibody
2. Anti-double-stranded DNA antibody
1. HLA-DR2
2. HLA-DR3
What is sarcoidosis?
A multisystem inflammatory disease characterized by the presence
of noncaseat- ing granulomas often found in the lungs and lymph nodes
but can be present in any or- gan system. Presenting symptoms are
often vague but may include arthralgias, muscle pains, and skin rash.
1. Diffuse scleroderma
2. CREST syndrome
What is polymyositis?
A chronic inflammatory myopathy which may present with proximal
muscle weakness. The etiology is not clearly known.
What is fibromyalgia?
A chronic syndrome involving diffuse pains affecting the entire body
and areas of tenderness in joints, muscles, and other soft tissues
What lab values should be evaluated in fibromyalgia?
CBC—normal; ESR—normal
Why is Kawasaki disease one of the only indications for using aspirin in
children?
The risk of developing Reye syndrome limits the use of aspirin in
children, a not- able exception is in the treatment of Kawasaki disease.
What is the other disease that involves both the respiratory tract
(hemoptysis) and kidney (renal failure)?
Goodpasture syndrome
Neoplastic
What is an enchondroma?
A benign cartilaginous neoplasm usually found in distal extremities
What is the most common primary malignant tumor derived from bone
(not bone marrow elements)?
Osteosarcoma
Infectious
What are some common organisms that are found uniquely in the joints of
infants and young children?
Group B streptococci; Haemophilus influenzae
Which serologic test detects syphilis earliest, is the most specific, and
stays positive even after treatment?
Fluorescent treponemal antibody absorption test (FTA-ABS)
What are the treatment options for syphilis in primary and secondary
stages?
Intramuscular penicillin G
What is the classic reaction occurring hours after treatment that
involves shaking chills, sore throat, myalgia, and malaise?
Jarisch-Herxheimer reaction
CLINICAL VIGNETTES
A 14-year-old football player is seen by his PCP for finger pain that
began when while in the process of catching a football there was violent
hyperextension of his distal in- terphalangeal (DIP). The patient is able
to extend at the joint, but is unable to flex it. What common injury is
this?
Jersey finger
A basketball player presents with pain in his finger that began when
the basketball struck his rigid finger on the distal tip causing forceful
hyperflexion at the DIP. Now the patient is able to flex normally, but
there is a decrease in extension. What common injury is this?
Mallet finger
A 13-year-old boy presents with left knee pain that is reproducible with
squatting or extending knee against resistance. There is edema over the
anterior tibial tuberosity. What is the diagnosis?
Osgood-Schlatter (traction apophysitis)
*Schla TTer has two Ts for Tibial Tuberosity
A 7-year-old white boy presents to clinic with a limp and some mild
groin pain. Ima- ging shows that there is avascular necrosis of the
femoral head. What is the most likely diagnosis?
Legg-Calvé-Perthes disease
CARDIOVASCULAR
How does the anatomy differ from normal in transposition of the great
arteries (TGA)? The aorta arises anteriorly from the right ventricle,
while the pulmonary artery (PA) arises from the left ventricle. The
aorta is posterior to the PA in a normal heart. This condition may also
be referred to as transposition of the great vessels (TGV) by some
sources.
What are the five congenital heart diseases that cause cyanosis (right-to-
left shunt) early in postnatal life?
1. Truncus arteriosus
2. TGA
3. Tri cuspid atresia
4. Tetralogy of Fallot
5. Total anomalous pulmonary venous connection (five words)
*5Ts—count them out on your hand
What are the three congenital heart diseases that cause a left-to-right
shunt and late cyanosis?
1. PDA
2. VSD
3. Atrial septal defect (ASD)
RESPIRATORY
What are the risk factors for infantile respiratory distress syndrome
(IRDS)?
Prematurity, male gender, maternal diabetes, and delivery by
cesarean section
What is the number one cause of death in infants age 1 month to 1 year?
Sudden infant death syndrome (SIDS)
GASTROINTESTINAL
The biopsy of the intestine reveals a lack of ganglion cells (in Auerbach
and Meissner plexuses) in the muscle wall. What is the mechanism of
this defect?
Failure of neural crest cell migration. These neural crest cells
eventually become ganglion cells.
An infant is born with the abdominal contents outside the body, yet
contained in a midline sac of peritoneum. What is the diagnosis?
Omphalocele
*The “O” reminds you of the belly button which is midline and
covered (by skin)
MUSCULOSKELETAL
What is the genetic defect in Duchenne muscular dystrophy?
X-linked (Xp21.2) deletion of the dystrophin gene
What is achondroplasia?
The most common form of dwarfism, resulting in shortened limbs
with relative preservation of trunk length. Other features include
macrocephaly, frontal bossing, and “trident” hand appearance.
What are the other small round blue cell tumors which may occur in
children?
Lymphoma; Neuroblastoma; Rhabdomyosarcoma; Ewing sarcoma;
Wilms tumor
*Remember Lyn Rhew knew tumors
NEUROLOGIC
What neural tube defect causes meninges and the spinal cord to herniate
through a spinal canal defect?
Meningomyelocele
What diet supplement has been shown to reduce the incidence of neural
tube defects?
Folic acid
• NF1—chromosome 17q
• NF2—chromosome 22q
CLINICAL VIGNETTES
You are making rounds in the neonatal intensive care unit. One infant
has rocker-bot- tom feet, low-set ears, micrognathia, a prominent
occiput, and clenched hands. What is the name and cause of the most
likely diagnosis?
Edwards syndrome—caused by Trisomy 18
A 7-year-old boy presents with his mother who states she cannot
control his appetite. She catches him eating food out of the trash cans
and she had to put a lock on the pantry. He has small extremities,
mental retardation, and microphallus. What is the name and cause of
the most likely diagnosis?
Prader-Willi syndrome—Paternal 15q11-13
A 2-year-old girl presents for her well-child check-up. You note she is
short in stature, has shortened limbs, frontal bossing, and slight
midface deficiency. What do you sus- pect?
Achondroplasia
A 14-year-old boy presents with left knee pain and swelling that has
increased over the month. On examination, you palpate a mass over
the tibia with warmth, tender- ness, and decreased range of motion in
the knee. What is the most likely diagnosis?
Osteosarcoma, the most common primary malignant tumor of the
bone in children
A 12-year-old boy presents with a painful mass on his thigh. You note
the area is tender, warm, and swollen. An x-ray shows a destructive
lytic tumor with surround- ing bone in an “onion-skin” appearance.
What is the most likely diagnosis?
Ewing sarcoma
A 4-year-old girl presents with high, spiking fevers for 3 weeks. She
also has a rash, body aches, and refuses to stand. You find she has
lymphadenopathy and joint swell- ing. What do you suspect?
Juvenile rheumatoid arthritis (JRA)
An 8-year-old girl previously diagnosed with attention-
deficit/hyperactivity disorder presents with continued attention
problems. Her mother states that several times a day, her daughter will
stare off into space for 5 to 10 seconds, become unresponsive, and have
eye fluttering. What do you suspect?
Absence seizures
A 7-year-old girl presents with persistent headaches over the past month
and recent onset of left-sided facial paralysis. What might you suspect?
Astrocytoma
A 5-year-old boy presents limping into your office with petechiae on his
face and chest. He has a temperature of 100.5°F and hepatomegaly.
What do you suspect?
Acute lymphoblastic leukemia (ALL)
A 3-year-old girl presents with a mediastinal mass and immature T cells.
What do you suspect?
Lymphoblastic lymphoma
CHAPTER 16
Radiology & Pathology Correlation
GENERAL PRINCIPLES
What are the five basic densities on a radiograph, from least to most
dense?
1. Air—least dense
2. Fat
3. Water (blood and soft tissue)
4. Bone
5. Metal—most dense
Why should you NOT use intravenous (IV) contrast when doing a CT
on a patient with a new head injury?
IV contrast can be confused with fresh blood in the brain.
What is ultrasonography?
A technique using high-frequency sound waves to make images
What are the two basic types of images in magnetic resonance (MR)?
1. T1-weighted images
2. T2-weighted images
What two modalities provide definitive imaging of the skull and brain?
1. CT
2. MRI
What should one suspect if there is a fracture over the middle meningeal
artery area?
Epidural hematoma
CHEST
What is bronchiectasis?
Dilatation of the bronchi, either diffuse or focal, often as a result of
chronic infec- tion and subsequent cartilage damage
What lung cancers more commonly arise centrally (near the hilum)?
Squamous cell and small cell carcinomas
What is the most valuable imaging method for staging lung cancers?
CT scan
CARDIOVASCULAR SYSTEM
1. Pulmonary stenosis
2. Pulmonary hypertension
3. Patent ductus arteriosus (PDA) or atrial septal defect (ASD) (due to
increased flow through the pulmonary artery)
What are the causes of acyanotic heart disease with normal pulmonary
vascularity?
Aortic stenosis, pulmonary stenosis, coarctation, and interruption of
the aortic arch
What should one look for in the case of acyanotic heart disease with
increased pul- monary vascularity?
Left atrial enlargement and a possible PDA or VSD
What is the most sensitive and specific study to evaluate for suspected
PE?
CT angiogram (CTA) is the gold standard
What causes aneurysms of the aortic arch and the descending thoracic
aorta?
The most common cause is atherosclerosis, but they can also result
from fibro- muscular dysplasia and cystic medial necrosis.
BREAST
What is the study of choice for breast imaging in women >40 years?
Mammography
What are the two views that mammograms are obtained in?
1. Craniocaudal
2. Axillary oblique views
How do the breasts of young women differ from those of older females?
Young women have extremely dense breast tissue, whereas older
women have more fatty tissue and atrophy of the breast parenchyma.
Why are mammograms not recommended for women under the age of
30?
Cancer is not easily detected by mammography in dense tissue.
GASTROINTESTINAL
What are the things one should look for on a plain abdominal film
(KUB)?
Gas pattern, organ shapes and sizes, calcifications, basilar lung
abnormalities, and skeletal abnormalities
What is the easiest way to identify small amounts of free air in the
peritoneal cavity?
Upright chest x-ray or KUB. Free air in the abdomen will rise and
be visible under the diaphragm.
What are the two imaging modalities that are often used to evaluate the
appendix?
True or false? About 90% of polyps are hyperplastic or only show low-
grade dysplasia (and are essentially benign):
True
What are the complications of acute pancreatitis and the study of choice
to rule out these complications?
Possible complications include pseudocyst and abscess formation,
which typically present 4 to 6 weeks after the original bout of
pancreatitis. A CT scan is the imaging modality of choice.
What are the two most common imaging techniques used to evaluate the
biliary sys- tem?
1. Ultrasonography
2. Endoscopy
GENITOURINARY SYSTEM
What is the most utilized imaging modality for the urinary tract and
what is it used to evaluate?
CT scan:
What is the most sensitive imaging modality for detecting renal stones?
CT scan
What are the primary imaging techniques used to evaluate the bladder?
Cystograms and CT scans
What is a cystogram?
A radiograph of the bladder
What type of study is needed when there is pelvic trauma that results in
injury to the urethra?
A retrograde urethrogram
A male patient comes to the ER with pelvic trauma and blood in the
urethral meatus. What must be done before catheterization of the
bladder can take place?
A retrograde urethrogram must be done to avoid enlarging a small
initial tear upon catheter entry.
Can PSA levels be used as a screening tool for men with possible prostate
cancer?
Given that PSA is neither sensitive nor specific, it is NOT an
optimal screening test. However, it is useful for checking overall trends
in PSA levels and following PSA levels posttreatment for prostate
cancer.
What are the most common lesions of the male external genitalia that
require ima- ging?
Epididymitis, testicular torsion (a medical emergency!),
hematoma, hydrocele, and testicular tumors
What study should be ordered when evaluating the testicle for either a
hydrocele or a tumor?
Testicular ultrasound
What is the most common imaging technique used to evaluate the female
pelvis?
Ultrasonography
SKELETAL SYSTEM
What is the most common utility of plain films of the skeletal system?
To evaluate for fracture
What is the role of CT scanning in the evaluation of the skeletal system?
It is useful for the evaluation of fine bone structure, particularly of
the skull, spine, and pelvis.
What is spondylolysis?
A term used to describe a break in the pars interarticularis of the
vertebral body
What is spondylolisthesis?
A term used to describe bilateral spondylolysis, when the vertebral
body slips for- ward on the vertebral body immediately below it
What are the three most common degenerative findings in the thoracic
spine?
1. Hypertrophic osteophytes
2. Calcification along the anterior spinal ligament
3. Calcification of an intervertebral disk
What are the common degenerative changes that occur in the lower
lumbar spine?
Herniated and protruded disks
What does subperiosteal, reactive new bone in Ewing sarcoma look like
on radio- graphy?
It looks like an “onion skin.”
If the periosteal reaction is located around a joint like the knee, what
kind of tumor should be suspected?
Osteosarcoma
What are the three classic forearm fractures requiring a forearm x-ray?
1. Nightstick fracture
2. Monteggia fracture
3. Galeazzi fracture
What is a chondrosarcoma?
A malignancy arising from the cartilage; it is the third most common
adult primary bone tumor (after multiple myeloma and osteosarcoma)
What is an osteochondroma?
A benign outgrowth of the bone that typically occurs in the lower
extremity. The cortex of the bone sticks out on a stalk and ends with a
bulbous cartilage cap.
What are the two most common soft tissue injuries of the knee?
PEDIATRICS
What are the most common conditions in children that require imaging?
Infections, trauma, and congenital abnormalities
What type of imaging is done when evaluating the fetal and infant brain?
Ultrasonography, as long as the fontanelles remain open
What are the two most common indications for ultrasound imaging?
1. Evaluation of ventricular enlargement (hydrocephalus)
2. Assessment of suspected brain hemorrhage
What is seen on lateral soft tissue view of the neck in acute epiglottitis?
A thickened epiglottis, often appearing bulbous and in the shape of a
“thumb.” Other findings include ballooning of the hypopharynx and
subglottic edema.
What is the major difference (besides size) between the chest x-ray of an
adult or child and that of a neonate?
The presence of the thymus, which is routinely identified on chest
films from birth to approximately 2 years of age
What are the lung findings seen on chest x-ray in transient tachypnea of
the newborn?
Lung volumes may be larger than normal, and there may be linear
or streaky opa- cities that clear within 2 days.
What are the lung findings seen on chest x-ray in a patient with neonatal
pneumonia?
The affected lung may be low in volume, normal, or hyperinflated.
Lung opacities are typically granular and the time course is variable.
1. Hydronephrosis
2. Vesicoureteral reflux
What is the initial imaging test of choice for evaluating hydronephrosis?
Abdominal ultrasound
What are the common midshaft fractures that occur in both children and
adults?
Transverse, oblique, spiral, and comminuted fractures
CLINICAL VIGNETTES
A patient presents with abdominal pain and states that he has not had
a bowel move- ment in 4 days. He also states that he is unable to sit due
to extreme pain in his anal region. What does this patient likely have
and what is the classic radiographic finding on barium study?
Crohn disease, with possible anal fistula; may see a string sign on
barium study. Crohn typically involves the distal ileum/proximal colon
and causes transmural in- flammatory changes. Its progression is
typically irregular (skip lesions) and can in- volve the whole GI tract.
a
c
h
o
n
d
r
o
p
l
a
s
i
a
acid-base
disorders,
diagnosis of acid-
base physiology
aci
d-
fast
stai
n
acn
e,
ind
ucti
on
of
aco
usti
c
neu
ro
ma
acquired immunodeficiency
syndrome (AIDS) acral-
lentiginous melanoma
acromegaly, growth hormone
adenoma with ACTH. See
adrenocorticotropic hormone
actinomyces israelii
acute anterior uveitis, HLA
allele and acute
inflammation
chara
cteriz
ation
of
outco
mes
of
vasoa
ctive
amine
s in
vasod
ilation
in
acute lymphoblastic leukemia (ALL)
acute myeloblastic
leukemia (AML) acute
pericarditis
acute poststreptococcal
glomerulonephritis acute
pyelonephritis
acute tubular
necrosis
(ATN) AD.
See autosomal
dominant
Addison
disease
adenocarcino
ma
adenomatous
polyps
adenosine deaminase
deficiency adenovirus
pneumonia
adre
nal
aden
oma
adre
nal
corte
x
adre
nal
glan
ds
adrenocorticotropic
hormone (ACTH) adult
cortex
adult respiratory distress syndrome
(ARDS) aflatoxin
AFP. See alpha-
fetoprotein
agranulocytosis, drugs and
AIDS. See acquired
immunodeficiency syndrome air
contrast enema
ALA. See δ-
aminolevulinic acid
alanine
transaminase (ALT)
Albright hereditary
osteodystrophy
alcohol abuse
dilated cardiomyopathy
and leukoplakia and
liver and
macrocyti
c anemia
and with
malnutriti
on
oral
cancer
and
alcoholi
c liver
disease
aldolas
eB
alkapto
nuria
ALL. See acute
lymphoblastic leukemia
allantois
a
l
l
e
l
e
a
l
l
o
g
r
a
f
t
alopecia
n
o
n
s
c
a
r
r
i
n
g
s
c
a
r
r
i
n
g
alpha-cell tumor
alpha-
fetoprote
in (AFP)
Alport
syndrom
e
ALS. See amyotrophic
lateral sclerosis ALT.
See alanine
transaminase alveolar
hypoxia
alveolar pressure, arterial
pressure and alveolar
volume (VA)
Alz
hei
mer
dis
eas
e
ami
no
aci
ds
δ-aminolevulinic acid (ALA)
AML. See acute
myeloblastic leukemia
amyloid deposits, stains
and amyotrophic lateral
sclerosis (ALS) analgesic
nephropathy
anaplasia
anaplas
tic
carcino
ma
anatom
ical
snuffb
ox
androg
en
insensi
tivity
anemia
, types
of
aneupl
oidy
Angel
man
syndro
me
angina
P
r
i
n
z
m
e
t
a
l
s
t
a
b
l
e
u
n
s
t
a
b
l
e
angiodysplasia,
of colon
angiogenesis,
mediators of
angiosarcoma
angiotensin-converting enzyme
(ACE) inhibitors anion gap acidosis
aniridia
ankylos
ing
spondyl
itis
ankyrin
annu
lar
panc
reas
anthr
acosi
s
anthrax
anti-HBc IgM. See antihepatitis B core
immunoglobulin M anti-HBsAg. See
antihepatitis B surface antigen antihepatitis
B core immunoglobulin M (anti-HBc IgM)
antihepatitis B surface antigen (anti-HBsAg)
antistreptolysin O (ASO) antibody titers
α
1
-
a
n
t
i
t
r
y
p
s
i
n
a
o
r
t
a
co
ar
cta
tio
n
of
tea
rs
of
traumati
c rupture
of aortic
dissection
aortic
regurgitati
on (AR)
aortic
stenosis
(AS)
apheresis
platelets
aphthous
ulcers
aplas
tic
anem
ia
apop
tosis
s
t
e
p
s
o
f
t
r
i
g
g
e
r
s
o
f
appen
dicitis
appen
dix,
cancer
of
appendix, carcinoid tumors and
AR. See aortic regurgitation;
autosomal recessive arcus corneae
ARDS. See adult respiratory
distress syndrome Argyll
Robertson pupils
Arnold-Chiari
malformation
arrhythmia
arterioscleros
is
arteriovenous
malformation
Arthus
reaction
AS. See aortic
stenosis
asbestosis
ascending aortic
dissection
Aschoff bodies
ash-leaf spots
ASO. See antistreptolysin O
antibody titers aspartate
transaminase (AST)
asperg
illosis
Asper
gillus
aspirat
ion
pneum
onia
AST. See aspartate
transaminase asthma
em
phy
se
ma
ma
nif
est
atio
ns
of
stat
us
ast
hm
atic
us
astro
cytes
astro
cyto
ma
atele
ctasi
s
ather
oscle
rosis
abdominal aortic
aneurysm and diabetes
mellitus and
ATN. See acute
tubular necrosis
atonic seizure
atopic
dermati
tis
atypical
pneumo
nia
Auspitz
sign
autoantibodies (AB), Hashimoto
thyroiditis and autograft
autoimmune hepatitis
autoimmune lymphocytic
adrenalitis autophagy
autopsy, myocardial
infarction and autosomal
dominant (AD)
inheritance
autosomal
recessiv
e (AR)
inherita
nce
azurophil granules
B
Babinski sign
bacilli, on
acid-fast
smear
Bacillus
anthracis
bacterial
endocarditi
s
bacteri
al
entero
colitis
bacteri
al
prostat
itis
bacteri
al
vagino
sis
Baker
cyst
b
a
l
a
n
i
t
i
s
b
a
m
b
o
o
s
p
i
n
e
b
a
r
i
u
m
e
n
e
m
a
B
a
r
r
b
o
d
i
e
s
Barr
ett
esop
hagu
s
Bart
holin
cyst
basal cell carcinoma, of skin
basic fibroblast growth
factor (bFGF)
basophilic stippling
b
a
t
t
l
e
s
i
g
n
B
e
c
k
t
r
i
a
d
Becker muscular
dystrophy (BMD)
Beckwith-
Wiedemann
syndrome Behçet
disease
benign nephrosclerosis
benign prostatic
hyperplasia (BPH)
benign tumors
Bernard-
Soulier
disease
Berry
aneurysms
bFGF. See basic fibroblast
growth factor bicornuate
uterus
bilateral
renal
agenesis
biliary
cirrhosi
s
biliary function,
laboratory values and
bilirubin
bilirubi
nemia,
types of
biotin
bite cells
bitemporal
hemianopsia
bladder
exstrophy
Blastomyces
dermatitidis
blastomycosis, location
occurrence of blindness
blood pH,
mechanoreceptors
and blue cell tumors,
small
BMD. See Becker
muscular dystrophy
Boerhaave tear
Borde
tella
pertus
sis
Borre
lia
burgd
orferi
Bouc
hard
nodul
es
bowel
obstru
ction
Bowe
n
diseas
e
Boxer
fractu
re
BPH. See benign
prostatic hyperplasia
brachial cleft cyst
b
r
a
d
y
k
i
n
i
n
b
r
a
i
n
b
r
e
a
s
t
s
cancer of
fibrocystic
changes/disease of
imaging of
Bren
ner
tumo
r
Bresl
ow
thick
ness
Broc
a
area
bromocriptine,
galactorrhea and
bronchial
carcinoid
bronchiectasis
condition
predisposition
of sputum
culture and
bronch
iolitis,
chroni
c
bronch
itis
c
h
r
o
n
i
c
p
r
e
s
e
n
t
a
t
i
o
n
o
f
bronchogenic carcinomas,
categories of bronze
diabetes
Brown-
Séquard
syndrome
bruise clue
color of
Brushfield
spots
Bubonic plague
Budd-
Chiari
syndrom
e
Buerger
disease
Buffalo
hump
bullo
us
pemp
higoi
d
burns
C
C
3
b
C
5
a
café au lait spots
CAH. See congenital adrenal
hyperplasia calcifications, breasts
and
calcified aortic stenosis,
mechanism of calcitonin
cal
ciu
m
sto
nes
Cal
l-
Ex
ner
bod
ies
Ca
mp
ylo
bac
ter
Campylobacter jejuni
cancer
o
f
a
p
p
e
n
d
i
x
o
f
b
r
e
a
s
t
s
cervical, human
papilloma virus and of
colon
endometrial
Epstein-
Barr
virus
and of
glans
penis
hepatoc
ellular
larynge
al
o
f
l
u
n
g
n
a
s
o
p
h
a
r
y
n
g
e
a
l
o
r
a
l
o
f
p
a
n
c
r
e
a
s
o
f
p
r
o
s
t
a
t
e
o
f
s
k
i
n
sta
gi
ng
sy
ste
m
of
ty
pe
s
of
i
n
c
h
i
l
d
r
e
n
i
n
m
e
n
i
n
w
o
m
e
n
w
e
i
g
h
t
l
o
s
s
a
n
d
cancer cachexia
Candida albicans
c
a
n
d
i
d
i
a
s
i
s
c
a
p
a
c
i
t
y
carbon monoxide, affect on oxyhemoglobin
dissociation curve carbon monoxide poisoning
carcinoi
d
syndro
me
carcino
ma in
situ
cardiac
tampon
ade
cardio
myopat
hy
d
i
l
a
t
e
d
h
y
p
e
r
t
r
o
p
h
i
c
r
e
s
t
r
i
c
t
i
v
e
carpal tunnel
syndrome
caseous
necrosis
c
a
t
f
e
c
e
s
c
a
t
a
r
a
c
t
s
c
e
l
i
a
c
s
p
r
u
e
c
e
l
l
s
of alveolar
system
apoptosis
a
t
r
o
p
h
y
c
h
i
e
f
c
h
r
o
m
a
f
f
i
n
d
e
a
t
h
o
f
f
o
a
m
G
a
u
c
h
e
r
h
y
p
e
r
p
l
a
s
i
a
h
y
p
e
r
t
r
o
p
h
y
i
n
j
u
r
y
t
o
irreversi
ble
mechani
sms of
reversib
le
L
e
y
d
i
g
l
i
n
i
n
g
m
e
t
a
p
l
a
s
i
a
mucosal
neck
multinucle
ate giant
necrosis
n
e
u
r
a
l
c
r
e
s
t
p
a
r
i
e
t
a
l
Sertoli
stres
s
adap
tatio
n of
Wart
hin-
Fink
elde
y
cellulitis
central
pontine
myelinolysis
centrilobar
emphysema
centrilobar
fibrosis
ceramide
trihexosidase
cerebral
arteries
cervi
cal
canc
er
cervi
cal
dyspl
asia
cervi
cal
poly
ps
cervi
citis
CF. See
cystic
fibrosis
Chagas
disease
chancre
chancroid
chemic
al
pneumo
nitis
chemor
eceptor
s
chemot
actic
agents
cherry
angiom
as
chest tube, tension
pneumothorax and
chicken pox
c
h
i
e
f
c
e
l
l
s
c
h
i
l
d
r
e
n
abuse of
b
ra
in
tu
m
o
rs
in
b
u
r
n
s
a
n
d
c
a
n
c
er
,
ty
p
e
s
o
f
herpes
gingivostomat
itis and iodine
deficiency in
tendonitis and
Chlamydia
trachomatis
infection
chocolate cysts
cholangiocarcino
ma cholecystitis
chole
lithia
sis
chon
drosa
rcom
a
chor
dee
choriocarci
noma
chromaffin
cells
chromoso
mal
disorders
chronic active hepatitis,
HLA allele and chronic
bronchitis
airflow
obstructio
n and
mucus
hypersecre
tion
chronic
gastritis
chronic
inflam
mation
causes
of
macrop
hages
monon
uclear
cells
and
chronic lymphocytic
leukemia (CLL) chronic
myelogenous leukemia
(CML)
chronic obstructive pulmonary
disease (COPD) chronic
restrictive pulmonary disease
chronic
rheumatic heart
disease Churg-
Strauss
syndrome
Chvostek sign
c
h
y
l
o
c
e
l
e
c
h
y
l
o
t
h
o
r
a
x
circulation, pulmonary
CLL. See chronic lymphocytic leukemia
Clostridiu
m difficile
Clostridiu
m
perfringen
s clot
lin
es
of
Za
hn
po
st
m
ort
e
m
cluster headache
CML. See chronic
myelogenous leukemia
CMV. See cytomegalovirus
coagulative necrosis
coal worker
pneumoconio
sis coarctation
of aorta
cocaine
, use of
Coccidi
oides
immitis
coccidi
oidomy
cosis
cold
sores
collagen
colle
cting
ducts
Colle
s
fract
ure
colo
n
canc
er
color
ectal
canc
er
compartment
syndrome
complete
abortion
compliance
computed
tomography
(CT)
condyloma
acuminatum
condylomata
lata
confabulation
congenital adrenal
hyperplasia (CAH)
congenital diaphragmatic
hernia congenital heart
diseases, cyanotic
congenital inguinal hernia
congenital
megacolon
congestive
heart
failure
Conn
syndrome
constrictiv
e
pericarditis
contact
dermatitis
COPD. See chronic obstructive
pulmonary disease Cori disease
coronar
y
angiogr
aphy
corpus
luteum
cysts
corticot
roph
Coryne
bacteri
um
cough
bro
nch
iect
asis
and
chr
oni
c,
cau
ses
of
noc
tur
nal
Cour
voisi
er
sign
Cow
den
synd
rome
coxs
ackie
virus
crani
al
nerv
es
crani
opha
ryngi
oma
crescentic
glomerulonephrit
is CREST
syndrome
cretinism
Creutzfeldt-Jakob disease
cri du
chat
syndrome
Crigler-
Najjar
syndrome
Crohn
disease
crou
p
cryo
preci
pitat
e
Cryptococcus neoformans
cryptorchidism
CT. See
computed
tomography
Cushing disease
Cushing
syndrome
cutaneous T-
cell
lymphoma
cyanide
poisoning
cyclophospha
mide
cystic fibrosis (CF)
with fat-soluble
vitamin deficiency
cysticercosis
c
y
s
t
i
n
e
s
t
o
n
e
s
c
y
s
t
i
n
u
r
i
a
c
y
s
t
i
t
i
s
cysts
B
a
k
e
r
B
a
r
t
h
o
l
i
n
c
h
o
c
o
l
a
t
e
c
o
r
p
u
s
l
u
t
e
u
m
dialysis-
associated
acquired
follicular
s
i
m
p
l
e
r
e
n
a
l
t
h
e
c
a
-
l
u
t
e
i
n
cytomegalovirus
(CMV)
infection
p
n
e
u
m
o
n
ia
c
y
t
o
t
o
x
ic
e
d
e
m
a
d
a
n
t
r
o
l
e
n
e
d
a
r
k
f
i
e
l
d
e
x
a
m
De Quervain
tenosynovitis
De Quervain
thyroiditis
dead space
volume (VD)
decubitus
films
degenerative joint
disease (DJD)
dementia
Denys-
Drash
syndrom
e
dermato
myositis
DES. See diffuse
esophageal spasm
desmoglein
desmosomes
DEXA. See dual energy x-ray
absorptiometry scan DI. See diabetes
insipidus
diabetes
insipidus (DI)
diabetes
mellitus type
I diabetes
mellitus type
II diabetic
ketoacidosis
(DKA)
diabetic
nephropathy
dialysis
dialysis-associated
acquired cysts diaphragm
diascopy
DIC. See disseminated intravascular
coagulation diffuse cortical necrosis
diffuse esophageal
spasm (DES)
diffuse large cell
lymphoma diffuse
pulmonary lung
disease DiGeorge
syndrome
dihydroxyphenylala
nine (DOPA)
diphyllobothrium
latum
disks, herniated
disseminated intravascular
coagulation (DIC) diverticulitis
diverticulosis
DJD. See
degenerative joint
disease DKA. See
diabetic ketoacidosis
DMD. See Duchenne
muscular dystrophy DNA.
See Hepadnaviridae
Donovan bodies
Donovania granulomatosis
DOPA. See
dihydroxyphenylalanine
double bubble sign
double uterus,
double vagina and
double vagina,
double uterus and
Down syndrome
dox
oru
bici
n
Dre
ssle
r
syn
dro
me
dual energy x-ray absorptiometry
(DEXA) scan Dubin-Johnson
syndrome
Duchenne muscular
dystrophy (DMD)
duodenal atresia
duplex
ultrason
ography
Dupuytr
en
contractu
re Duret
hemorrh
ages
dysgerm
inoma
dysplasi
a
dysplasti
c nevus
dyspnea,
positiona
l
dystrophi
c
calcificat
ion
v
a
s
o
g
e
n
i
c
Edwards
syndrome
Ehlers-
Danlos
syndrome
Eisenmen
ger
syndrome
electrolyte
abnormali
ties
11q13
embolic pulmonary
arterial occlusion embryo,
genotype of
embryonal
carcinoma
emphysema
airspace
enlargeme
nt alveolar
wall
destructio
n
centrilobul
ar
p
a
n
a
c
i
n
a
r
p
r
e
s
e
n
t
a
t
i
o
n
o
f
emphysematous cystitis
emphysematous lung disease,
autopsy and empty sella
syndrome
empyema
encephalotrigeminal
angiomatosis encysted
bradyzoites
endod
erm
endod
ermal
sinus
endog
enous
pigme
nts
endometrial
cancer, obesity
and endometrial
carcinoma
endometrial
hyperplasia
endometrial
polyps
endometriosis
endometritis
end-
stage
liver
disease
end-
stage
nephro
pathy
enema
a
i
r
c
o
n
t
r
a
s
t
b
a
r
i
u
m
Entero
bacter
cloaca
e
entero
virus
epider
mis
epidermolysis bullosa
acquisita (EBA)
epididymitis
epid
ural
hem
ato
ma
epig
lotti
s
epis
padi
as
epithelial origin, surface
epith
eli
u
m
es
op
ha
gu
s
an
d
of
lar
ge
int
es
tin
e
si
m
pl
e
cu
bo
id
al
of
s
m
all
int
es
tin
e
st
o
m
ac
h
an
d
strati
fied
squa
mou
s
trans
ition
al
Epstein-Barr
virus
(EBV)
cancer and
le
uk
op
la
ki
a
an
d
tu
m
or
s
an
d
Erb-Duchenne paralysis
ERV. See expiratory
reserve volume
erythema chronicum
migrans erythema
infectiosum
erythema
marginat
um
erythema
multifor
me
erythras
ma
erythrobl
astosis
fetalis
erythropl
asia
Escheric
hia coli
esophage
al atresia
esophagi
tis
esophago
gram
esophagu
s
e
p
i
t
h
e
l
i
u
m
a
n
d
l
a
y
e
r
s
o
f
m
u
s
c
l
e
s
o
f
squamous cell
carcinoma of
tears of
estrogen
ethanol, methanol
poisoning and Ewing
sarcoma
exanthema
subitum
exogenous
pigments
exogenous
steroids
expiratory reserve volume
(ERV) extramammary
Paget disease
extrinsic allergic alveolitis
extrinsic (death receptor-
initiated) pathway exudate
exudative pleural effusion
Fabry disease
fallopian tubes, tumors of
familial adenomatous
polyposis (FAP)
familial adenomatous
polyposis coli familial
cancer syndrome
familial goiter
skin hamartomas
familial
hypercholesterol
emia Fanconi
syndrome
FAP. See familial
adenomatous polyposis
Farmer lung
f
a
s
c
i
o
t
o
m
y
f
a
t
e
m
b
o
l
i
fatty
acid
synthesi
s fatty
streaks,
in
vessels
FBN-1.
See
fibrillin-
1 Felty
syndro
me
femoral
neck
fracture
ferrugin
ous
bodies
fetal
alcohol
syndro
me fetal
aplastic
anemia
fetal
cortex
fever blisters
FFP. See fresh
frozen plasma
fibrillin gene
fibrillin
-1
(FBN-
1)
fibrinoi
d
necrosi
s
fibrinou
s
pericard
itis
fibroad
enoma
fibrocystic disease,
histologic types of
fibroma
fibromus
cular
dysplasia
fibromya
lgia
fi
br
ou
s
dy
sp
la
si
a
fif
th
di
se
as
e
fine needle aspiration
(FNA), of thyroid first-
degree burn
Flaviviridae (RNA)
fluorescence treponemal antibody-absorption test (FTA-ABS)
FNA. See fine
needle aspiration
foam cells
focal segmental
glomerulosclerosis
folic acid
follic
ular
carci
noma
follic
ular
cysts
foreign
body
granulom
a
foreskin
f
r
a
c
t
u
r
e
s
b
e
n
d
i
n
g
b
o
w
i
n
g
b
u
c
k
l
e
c
l
a
v
i
c
u
l
a
r
C
o
l
l
e
s
epiphyse
al-
metaphy
seal
femoral
neck
greenstic
k
h
a
n
g
m
a
n
o
l
e
c
r
a
n
o
n
o
f
s
c
a
p
h
o
i
d
of
th
or
aci
c
spi
ne
tor
us
fragile X syndrome
FRC. See functional
residual capacity free
tachyzoites
fresh
frozen
plasma
(FFP)
fructokinas
e
fructose
FTA-ABS. See fluorescence treponemal antibody-
absorption test functional residual capacity (FRC)
G
s
p
e
c
i
f
i
c
granuloma
inguinale
granulomas
f
o
r
e
i
g
n
b
o
d
y
i
m
m
u
n
e
t
y
p
e
s
o
f
granulo
matous
arteritis
granulos
a cell
tumors
Graves
disease
gray
baby
syndr
ome
gray
matter
greens
tick
fractu
re
group
A
strept
ococci
growth hormone adenoma,
with acromegaly GSP
oncogene
gubernaculum
Guillain-Barré syndrome
GVHD. See graft-versus-host disease
gynecomastia, drugs and
Haemop
hilus
ducreyi
haemoph
ilus
influenza
e
Hagema
n factor
h
a
i
r
f
o
l
l
i
c
l
e
s
h
a
m
a
r
t
o
m
a
hand-foot-
and-mouth
disease
hangman
fracture
Hartnu
p
disease
Hashi
moto
thyroid
itis Hb
Barts
HBsAg. See hepatitis B
surface antigen HCG. See
human chorionic
gonadotropin headaches
bacte
rial
meni
ngiti
s
types
of
heart, metastatic
tumor and
Heberden
nodules
Hein
z
bodi
es
Heli
coba
cter
pylor
i
helio
trope
rash
HELLP syndrome (hemolysis, elevated liver function tests, low
platelets) hematocele
he
ma
toc
rit
he
mi
des
mo
so
me
s
he
mi
zy
go
us
he
mo
chr
om
ato
sis
he
mo
glo
bin
hemolysis, elevated liver function tests, low platelets (HELLP
syndrome) hemolytic uremic syndrome (HUS)
hemo
philia
,
classi
c
hemo
rrhag
ic
cystit
is
hemo
rrhoi
ds
hemo
sideri
n
hemothorax
Henderson-
Patterson
bodies
Henoch-
Schönlein
purpura
Hepadnaviri
dae (DNA)
hepatic
adenomas
h
e
p
a
ti
c
f
a
il
u
r
e
h
e
p
a
ti
c
n
e
c
r
o
s
i
s
h
e
p
a
ti
ti
s
A
hepatitis B infection
hepatitis B surface antigen
(HBsAg) hepatitis C
h
e
p
at
it
is
D
h
e
p
at
it
is
E
h
e
p
at
it
is
v
ir
u
s
e
s
hepatobiliary iminodiacetic acid
(HIDA) scan hepatocellular cancer
hepatocyte function, laboratory
values and hereditary fructose
intolerance
hereditary hemorrhagic
telangiectasia hereditary
nonpolyposis colon cancer
(HNPCC) hereditary
spherocytosis
he
r
m
ap
hr
od
iti
s
m
he
rn
ia
congenital
diaphragmatic
congenital
inguinal
herpes encephalitis
herpes
simplex
virus
(HSV)
pneumonia
tra
ns
mi
ssi
on
of
ty
pe
2
her
pes
zost
er
her
peti
c
sto
mat
itis
het
ero
pha
gy
het
ero
zyg
ous
HGPRT. See hypoxanthine-guanine
phosphoribosyltransferase HHV 8. See human
herpes virus 8
HIDA scan. See hepatobiliary
iminodiacetic acid scan Hill-Sachs
deformity
Hirschsprung disease
h
i
s
t
a
m
i
n
e
h
i
s
t
i
o
c
y
t
e
h
i
s
t
i
o
c
y
t
o
s
i
s
X
Histoplasma capsulatum
histoplasmosis
HIV. See human
immunodeficiency virus
HLA. See human
leukocyte antigen HMD.
See hyaline membrane
disease
HNPCC. See hereditary
nonpolyposis colon cancer
hoarseness
Hod
gkin
disea
se
Hod
gkin
lymp
hom
a
hom
ozyg
ous
Horn
er
synd
rome
horse
shoe
kidn
ey
hous
emai
d
knee
HSV. See herpes
simplex virus
HTN. See
hypertension
human chorionic
gonadotropin (hCG)
human herpes virus 8
(HHV 8)
human immunodeficiency virus
(HIV) meningoencephalitis
oral thrush and
human leukocyte
antigen (HLA)
human papilloma
virus (HPV)
cervical
carcino
ma and
oral
cancer
and
verruca
vulgaris
and
Hun
ter
synd
rom
e
Hun
tingt
on
dise
ase
Hurl
er
synd
rom
e
HUS. See hemolytic
uremic syndrome
hyaline membrane
disease (HMD)
hyalinized collagen
hydatidif
orm
mole
hydrocel
e
hydrocep
halus,
types of
hydronep
hrosis
21-hydroxylase deficiency
hyperald
osteroni
sm
hypercal
cemia
hypercal
ciuria
hyperco
agulable
state
hyperkal
emia
hyperker
atosis
hyperlip
idemia
hyperpar
athyroid
ism
hyperpig
mentatio
n
hyperpla
sia
hyperse
nsitivity
angitis
hypersensitivi
ty
pneumonitis
hypersensitivi
ty reactions
hypertension
(HTN)
brain
parenc
hyma
and
causes
of
compli
cations
of
emerge
ncy
m
a
l
i
g
n
a
n
t
p
o
r
t
a
l
r
i
s
k
f
a
c
t
o
r
s
f
o
r
u
r
g
e
n
c
y
hyperthyroidis
m
hypertrophic
cardiomyopath
y hypertrophic
pyloric
stenosis
hypertrophy
hypo
calce
mia
hypo
parat
hyroi
dism
hypo
spadi
a
hypo
spadi
ac
ureth
ra
hypot
hala
mus
hypot
hyroi
dism
hypoxanthine-guanine phosphoribosyltransferase
(HGPRT) hypoxemia, pulmonary causes of
hypoxia
hypoxic pulmonary
vasoconstriction
hypsarrhythmia
hysterosalpingogram
I
c
h
r
o
n
i
c
inflammatory bowel
disease (IBD) aphthous
ulcers and
inflammatory exudate, removal of
influenza
pneumoni
a
inspirator
y
capacity
(IC)
insulin
synthesis
insulinom
a
intersexu
ality
interstitial
nephritis
intestinal
lymphangiec
tasia
intestines
o
b
s
t
r
u
c
t
i
o
n
o
f
s
m
a
l
l
absorptive capacity
increase in function
of
tumors of
intracellular
accumulations, types of
intracranial pressure (ICP)
intracytoplasmic Reinke
crystals intraductal
papilloma
intravenous drug users,
endocarditis and intrinsic
(mitochondrial) pathway
intussusception
iodine deficiency, in children
IPF. See idiopathic
pulmonary fibrosis iritis
ischemia
, injury
ischemic
bowel
disease
ischemic
colitis
ischemic
stroke
i
s
l
e
t
c
e
ll
t
u
m
o
r
s
i
s
o
c
h
r
o
m
o
s
o
m
e
ITP. See immune
thrombocytic purpura
Ixodes tick bites
J
a
c
k
s
o
n
i
a
n
J
a
n
e
w
a
y
l
e
s
i
o
n
s
Jarisch-Herxheimer reaction
j
a
u
n
d
i
c
e
j
e
r
s
e
y
f
i
n
g
e
r
JRA. See juvenile
rheumatoid arthritis
juvenile polyps
juvenile rheumatoid arthritis (JRA)
Kaposi
sarco
ma
Kartag
ener
syndro
me
karyor
rhexis
Kawas
aki
diseas
e
keloid
keratoconj
unctivitis
sicca
Kerley B
lines
kernicteru
s
kidneys, blood
supply of
kidneys, ureter,
bladder (KUB)
Kimmelstiel-
Wilson nodules
kinin system
klebsiell
a
pneumon
iae
Klinefelt
er
syndrom
e
Klüver-
Bucy
syndrom
e
Koebner
phenome
non
KOH
preparati
on
koilocyt
osis
Koplik
spots
Korsak
off
psycho
sis
Krabbe
disease
Kruken
berg
tumor
KUB (kidneys,
ureter, bladder)
Kuru plaques
Kuss
maul
breat
hing
Kwas
hiork
or
L
lacta
se
defic
iency
lacto
troph
LAD. See left anterior
descending Landsteiner
rule
Langerhans cell
histiocytosis
Langerhans cells
Large
cell
carcin
oma
laryng
eal
cancer
lateral
epicon
dylitis
lead
poison
ing
leather bottle stomach
left anterior
descending (LAD)
artery Legg-Calvé-
Perthes disease
Legionella
pneumophila
leio
myo
ma
leio
myos
arco
mas
lentigo
maligna
melanoma
leprosy
Lesch-
Nyhan
syndrom
e leucine
leukemia
leukocyte extravasation,
steps of leukoplakia
Leyd
ig
cell
tumo
rs
Leyd
ig
cells
LH. See
luteinizing
hormone
Libman-Sacks
endocarditis
lichen
sclerosus
lichen simplex
chronicus
lichenification
l
i
g
h
t
c
r
i
t
e
r
i
a
l
i
n
e
s
o
f
Z
a
h
n
l
i
n
i
n
g
c
e
l
l
s
l
i
n
i
t
i
s
p
l
a
s
t
i
c
a
liquefactive necrosis
Lisch nodules
lithium
exposure,
chronic
Littré glands
liver
alc
oh
ol
ab
use
an
d
fatt
y
ch
an
ge
in
fun
cti
on
s
of
nut
me
g
pre
gn
an
cy
an
d
reg
en
era
tio
n
of
Loeffler obliterative
cardiomyopathy long
bone fracture, fat
emboli and Lou
Gehrig disease
lu
m
b
ar
p
u
n
ct
ur
e
lu
n
g
s
abscess of
clinica
l signs
of
organi
sm
causin
g
radiog
raphic
signs
of
treatm
ent of
b
e
n
i
g
n
l
e
si
o
n
o
f
b
l
o
o
d
fl
o
w
i
n
c
a
n
c
e
r
o
f
ad
en
oc
ar
ci
no
m
a
co
m
pli
ca
tio
ns
of
lar
ge
ce
ll
m
et
ast
asi
s
of
pl
eu
ra
m
et
ast
asi
s
s
m
all
ce
ll
s
m
ok
in
g
an
d
sq
ua
m
ou
s
ce
ll
function
of, aging
and
perfusion
distributio
n in
ventilation
distributio
n in
volumes
of
lupus
nephropath
y
luteinizing
hormone
(LH) Lyme
disease
lymphoblas
tic
lymphoma
lymphocytic
pleocytosis
lymphocytosis
lymphogranulom
a venereum
Lynch syndrome
l
y
o
n
i
z
a
t
i
o
n
l
y
s
i
n
e
m
a
c
r
o
p
h
a
g
e
s
m
a
c
u
l
e
magnesium ammonium phosphate
(struvite) stones magnetic resonance
imaging (MRI)
malabsorpt
ion
syndromes
Malassezia
furfur
malignant
mesothelio
ma
malignant tumors,
characteristics of mallet
finger
Mall
ory-
Wei
ss
tear
ma
mmi
llary
bodi
es
ma
mm
ogra
phy
MAOI. See monoamine
oxidase inhibitor MAP. See
mean arterial pressure
maple
syrup urine
disease
marasmus
marc
h
fract
ure
Marf
an
syndr
ome
massi
ve
hemo
ptysi
s
McA
rdle
disea
se
McA
rdle
syndr
ome
mean arterial
pressure (MAP)
measles
measles
pneumo
nia
mechan
orecept
ors
Meckel
divertic
ulum
meconi
um
aspirati
on
medial
epicond
ylitis
medial
lemniscal
pathway
mediastinal
silhouette
medullary
carcinoma
medulloblast
omas
Meig
s
syndr
ome
meiot
ic
divisi
on
Meis
sner
corpu
scles
mela
nin
m
e
l
a
n
o
m
a
m
e
l
a
s
m
a
membranoproliferative glomerulonephritis
(MPGN) membranous glomerulonephritis
membranous urethra
MEN. See multiple
endocrine neoplasia men,
cancer in
meni
ngio
ma
meni
ngiti
s
meni
ngoc
ele
meni
ngo
myel
ocele
meningovascular
neurosyphilis
Merkel cell
carcinoma
Merkel
corpuscles
mesonephric
ducts
mesonephric
tubules
metabolic
acidosis
metabolic
alkalosis
metachromic
leukodystrophy
metanephric
kidneys
metanephros
m
e
t
a
p
h
a
s
e
m
e
t
a
p
l
a
s
i
a
metastatic
calcification
methanol
poisoning
MG. See
myasthenia
gravis MI. See
myocardial
infarction
microglia
microsco
pic
polyangii
tis
migraines
Mikulicz syndrome
m
i
l
k
e
r
n
o
d
u
l
e
s
m
i
n
e
r
s
coal workers’
pneumoconiosi
s silicosis and
minimal
change
disease
missed
abortion
missense
mutation
mitral
regurgitation
(MR) mitral
stenosis
(MS)
mitral valve
prolapse
(MVP)
molluscum
contagiosum
monoamine oxidase
inhibitor (MAOI)
monoclonal neoplasm
mononuclear cells, chronic
inflammation and mononucleosis
Monte
ggia
fractur
e
Morax
ella
catarr
halis
morbil
liform
rash
motor neuron, upper versus lower
MPGN. See membranoproliferative
glomerulonephritis MR. See mitral
regurgitation
MRI. See magnetic
resonance imaging MS.
See mitral stenosis
mucinous
cystadenocarcino
ma mucinous
cystadenoma
mucocele
mucoepidermoid
carcinoma mucor
Mu
cora
les
mu
cos
al
nec
k
cell
s
Mül
leri
an
duc
ts
Müllerian-
inhibiting
factor
multinucleat
e giant cells
multiple endocrine
neoplasia (MEN) multiple
myeloma
multi
ple
scler
osis
mum
ps
Münchha
usen
syndrome
murmurs
Murphy sign
MVP. See mitral valve
prolapse myasthenia
gravis (MG)
Mycobacterium avium-
intracellulare
Mycobacterium leprae
Mycobacterium
tuberculosis
Mycoplasma
pneumoniae
mycosis
fungoides
myocardial
infarction
(MI)
arr
hy
th
mi
a
an
d
au
to
ps
y
an
d
ch
an
ge
s
po
st
coagulativ
e necrosis
and
fibrinous
pericarditi
s
macropha
ges and
neutrophil
s and
s
i
l
e
n
t
s
u
b
e
n
d
o
c
a
r
d
i
a
l
t
r
a
n
s
m
u
r
a
l
myositis
ossificans
myotonic
dystrophy
myxoma
tumors
NADPH
oxygenase,
deficient in
nasopharyngeal
cancer navicular
fossa
NEC. See
necrotizing
enterocolitis
necrosis
c
a
s
e
o
u
s
c
o
a
g
u
l
a
t
i
v
e
l
i
q
u
e
f
a
c
t
i
v
e
s
u
r
g
i
c
a
l
necrotizing enterocolitis
(NEC) necrotizing fascitis
necrotizing papillitis
Negri
bodies
Neisseri
a
gonorrh
ea
Neisseri
a
meningi
tides
Nelson
syndro
me
nephriti
c
syndro
me
nephrob
lastoma
nephroc
alcinosi
s
nephrog
enic
cord
nephron
ophthisi
s
nephrot
ic
syndro
me
neural
crest
cells
neurobl
astoma
neuroendocrine
system cells
neurofibromatosi
s (NF) neuromas
n
e
u
r
o
s
y
p
h
i
l
i
s
n
e
u
t
r
o
p
h
i
l
s
NF. See
neurofibromatosi
s niacin
nicotini
c acid
Niema
nn-
Pick
disease
Niemann-Pick
disease type C
nightstick
fracture
Nikolsky sign
nitroglycerin, stable
angina and nitrosamines
N-myc
Nocard
ia
asteroi
ds
nodular
melano
ma
nonani
on gap
acidosi
s
nonger
m cell
tumors
nonsen
se
mutatio
n
nuclear
imagin
g
null
-cell
ade
nom
as
nurs
ema
id
elbo
w
nut
meg
liver
O
obesity
diabetes
mellitus type II
and
endometrial
cancer and
hypertension
and
obliterati
ve
endarterit
is
obstructi
ve lung
disease
obstructi
ve
uropathy
ochronos
is
olecrano
n bursitis
oligoden
drocytes
oligohyd
ramnios
omphalo
cele
ophthal
mia
neonator
um
opioids
o
p
t
i
c
c
h
i
a
s
m
o
r
a
l
c
a
v
i
t
y
cancer of
stratified squamous
epithelium and oral
thrush
orchitis
organophosp
hate
poisoning
ornithosis
Orpha
n
Annie
nuclei
orthop
nea
Osgood-
Schlatter
disease
Osler
nodes
Osler-Weber-
Rendu syndrome
osmotic fragility
test
osteitis
fibrosa
cystica
osteoar
thritis
osteoblastic lesions,
indicator of
osteochondroma
osteogenesis imperfecta
type I osteoporosis
osteosarcoma
o
v
a
r
i
e
s
o
v
u
l
a
t
o
r
y
c
y
c
l
e
oxygen, increased
demand of oxygen-
dependent
phagocytosis
oxygen-
independent
phagocytosis
oxyhemoglobin
dissociation curve
oxytocin, effects of
P
p53 gene
paci
nian
corp
uscle
s
Page
t
disea
se
PAN. See
polyarteritis
nodosa
panacinar
emphysema
pancake kidney
P
a
n
c
o
a
s
t
t
u
m
o
r
p
a
n
c
r
e
a
s
pancreatic
cholera. See
VIPoma
pancreatic
pseudocyst
pannus
papil
lary
carci
noma
papu
le
parafollicu
lar C cells
parainfluen
za
pneumonia
parameson
ephric
ducts
paraneopla
stic
syndromes
paraphimo
sis
parathyroid
adenoma
parathyroid
glands
parathyroid
hormone
(PTH)
paravaccinia
virus
pareti
c
neuro
syphil
is
pariet
al
cells
Parkin
son
diseas
e
paroti
d
gland
tumor
s
paroxysmal nocturnal dyspnea
p
a
r
v
o
v
ir
u
s
B
1
9
P
a
t
a
u
s
y
n
d
r
o
m
e
patent ductus
arteriosus (PDA)
pathology,
definition of
pauciarticular juvenile
rheumatoid arthritis PCBs.
See polychlorinated
biphenyls PCOS. See
polycystic ovarian syndrome
PCR. See polymerase chain
reaction
PDA. See patent
ductus arteriosus
pellagra
pelvic inflammatory
disease (PID) pelvic
kidney
pemphigus
vulgaris
penicillin G,
syphilis and
peptic ulcer
disease
(PUD)
pericardial
effusion
ca
us
es
of
m
ali
gn
an
cy
an
d
peric
ardia
l
wind
ow
peric
ardit
is
a
c
u
t
e
c
a
u
s
e
s
o
f
c
o
n
s
t
r
i
c
t
i
v
e
periosteal hematomas
peripheral neuropathy, diabetes
mellitus and peritonitis
Peutz-Jeghers
polyposis syndrome
Peyronie disease
PGE. See
prostaglan
din E
phagocytos
is, stages
of Phalen
sign
phenotypic sexual
differentiation
phenylalanine
phenylke
tonuria
(PKU)
phenytoi
n
pheochr
omocyto
ma
phimosis
photosensitivity
reactions, drugs and
Phren sign
phyllod
es
tumor
Picorna
viridae
(RNA)
PID. See pelvic
inflammatory disease
Pierre Robin syndrome
pilocyt
ic
astroc
ytoma
pitcher
s
elbow
pituita
ry
a
n
t
e
r
i
o
r
h
o
r
m
o
n
e
s
pit
uit
ary
ade
no
ma
pit
uit
ary
apo
ple
xy
pit
uit
ary
gla
nd
PKD. See polycystic
kidney disease PKU.
See phenylketonuria
plac
enta
accr
eta
plac
enta
prev
ia
plac
enta
l
abru
ptio
n
co
caine
use
and
plain
radio
grap
hs
plaq
ues
platelets
pleomor
phic
adenom
as
pleural
exudate
Plumme
r
syndrom
e
Plummer-Vinson syndrome
PML. See progressive multifocal
leukoencephalopathy pneumoconioses
Pneumocystis carinii
pneumonia
pneumocytes type I/II,
alveolar system
pneumonia
bacterial
morphologic
patterns of
plague
s
i
g
n
s
o
f
s
y
m
p
t
o
m
s
o
f
v
i
r
a
l
pneumot
horax
poisonin
g,
antidotes
for polar
bodies
polio virus
polyarteritis
nodosa (PAN)
polychlorinated
biphenyls (PBCs)
polyclonal
neoplasm
polycystic kidney
disease (PKD)
polycystic ovarian
syndrome (PCOS)
polygenic disorder
polymerase chain reaction (PCR), herpes simplex
virus and polymyalgia rheumatica
p
o
l
y
m
y
o
s
i
t
i
s
p
o
l
y
p
l
o
i
d
y
P
o
m
p
e
d
i
s
e
a
s
e
portal
hypertension
portosystemi
c shunting
positive
pressure
ventilation
posterior
pituitary
postgonococcal arthritis,
HLA allele and postpolio
syndrome
postrenal azotemia
poststreptococcal
glomerulonephritis
Potter sequence
P
ot
te
r
s
y
n
dr
o
m
e
p
o
x
vi
ru
s
Prader-
Willi
syndrom
e
preecla
mpsia
pregnan
cy
ectopic
fat
ty
liver
and
prep
atell
ar
bursi
tis
prere
nal
azote
mia
preti
bial
myx
edem
a
priap
ism
prim
ordia
l
follic
les
primor
dial
germ
cells
Prinz
metal
angina
proger
ia
progressive multifocal leukoencephalopathy
(PML) prolactinoma
pronephros
prostagl
andin E
(PGE)
prostate
cancer
prostate-specific
antigen (PSA)
prostatic urethra
proteus mirabilis
prototypic acute restrictive (interstitial) lung
disorders Prussian blue dye, ferruginous
bodies
PSA. See prostate-
specific antigen
psammoma bodies
pseudo
intersexuality
pseudohermaphr
oditism
pseudohypoparat
hyroidism
pseudomembran
e formation
pseudomembran
ous candidiasis
pseudomembran
ous colitis
Pseudomonas
aeruginosa
psoriasis
psoriatic arthritis
PTH. See
parathyroid
hormone
puberty
PUD. See
peptic ulcer
disease
pulmonary
agenesis
congenital
diaphragmatic hernia and
pulmonary alveolar
proteinosis pulmonary
edema
pulmonary
emboli
pulmonary
embolus
pulmonary
fibrosis
pulmonary
gas exchange
pulmonary
hemorrhage
syndromes
pulmonary
hypertension
p
r
i
m
a
r
y
s
e
c
o
n
d
a
r
y
pulsel
essnes
s
diseas
e
pulsus
parad
oxus
pus
p
y
e
l
o
n
e
p
h
r
i
t
i
s
p
y
k
n
o
s
i
s
p
y
l
o
r
i
c
s
t
e
n
o
s
i
s
pyloromyotomy
pyruvate dehydrogenase deficiency
q-fever
RA. See
rheumatoid
arthritis
rabies virus
rapid plasma
reagent (RPR)
RBCs. See red
blood cells
RCC. See renal
cell carcinoma
RDS. See respiratory
distress syndrome rectal
bleeding
red
blood
cells
(RBCs)
red man
syndro
me
Reed-
Sternbe
rg cell
reflex sympathetic
dystrophy (RSD)
Reiter disease
Reiter
reactive
arthritis
Reiter
syndrome
ren
al
ade
no
ma
ren
al
age
nes
is
ren
al
am
ylo
ido
sis
ren
al
ang
ioli
po
ma
renal cell
carcinoma
(RCC) renal
failure
renal
papillary
necrosis
renal
stones
renal tubular
acidosis (RTA)
renal
ultrasonography
renin-angiotensin system,
activation of reperfusion
injury
reproductive system, development of
residual
volume (RV)
respiratory
acidosis
respiratory
alkalosis
respiratory distress
syndrome (RDS)
respiratory infections,
bacteria causing
respiratory muscles
a
c
c
e
s
s
o
r
y
d
i
a
p
h
r
a
g
m
n
e
r
v
e
s
a
n
d
respiratory syncytial virus
(RSV) bronchiolitis restrictive
lung disease
causes of
changes
occurring in
complications
of
RET
oncogen
e
retinobl
astoma
retrogra
de
urethrog
ram
Reye
syndrom
e
RF. See
rheumatoid
factor Rh+
rhabdomyomas
rhabdomy
osarcoma
rheumatic
fever
rheumatoi
d arthritis
(RA)
rheumatoi
d factor
(RF)
rickets
rickett
sia
akari
rickett
sia
prowa
zekii
rickett
sia
rickett
sii
rickett
siae
Ri
ed
el
th
yr
oi
dit
is
Ri
gl
er
si
gn
RNA. See Flaviviridae;
Picornaviridae RNA virus
RNA virus-like
caliciviruses
Robertsonian
translocation
Rocky Mountain
spotted fever
rosacea
Rose
nthal
fiber
s
rotat
or
cuff
injur
y
Roth
spots
rotor syndrome
RPR. See rapid plasma reagent
RSD. See reflex sympathetic dystrophy
RSV. See respiratory syncytial virus
bronchiolitis RTA. See renal tubular
acidosis
ru
be
lla
ru
di
m
en
tar
y
ho
rn
rul
e
of
2s
Russell bodies
RV. See residual volume
S
t
e
s
t
s
c
h
w
a
n
n
o
m
a
SCID. See severe combined immunodeficiency
disease scleroderma
SCPE. See slipped capital femoral
epiphysis scrofula
scurvy
sebo
rrhei
c
kerat
osis
seco
nd-
degr
ee
burn
seizu
res
a
b
s
e
n
c
e
a
t
o
n
i
c
r
a
d
i
o
g
r
a
p
h
y
f
o
r
s
i
m
p
l
e
p
a
r
t
i
a
l
t
o
n
i
c
-
c
l
o
n
i
c
s
e
m
i
n
o
m
a
s
e
n
il
e
a
n
g
i
o
m
a
s
senile
lysosomal
enzyme
septic
arthritis
septic joint
septicemia, blood
cultures and
serotonin
serous
cystadenoca
rcinoma
serous
cystadenom
a serratia
marcescens
Serto
li
cell
tumo
rs
Serto
li
cells
Sertoli-
Leydig cell
tumors
serum
calcium
serum electrolyte abnormalities
severe combined immunodeficiency disease
(SCID) sexually transmitted disease (STD)
Sézary
syndro
me
shaken
baby
syndro
me
Sheehan
syndro
me
Shilling
test
s
h
i
n
g
l
e
s
s
h
o
c
k
s
h
u
n
t
i
n
g
SIADH. See syndrome of inappropriate antidiuretic
hormone sialadenitis
sic
kle
cel
l
ane
mi
a
sic
kle
cel
l
dis
eas
e
SIDS. See sudden infant
death syndrome silent
mutation
s
i
l
i
c
o
s
i
s
s
i
l
v
e
r
s
t
a
i
n
simple
columnar
epithelium
simple
cuboidal
epithelium
simple
partial
seizure
simple renal
cysts
Sip
ple
syn
dro
me
situ
s
inv
ers
us
sixt
h
dis
eas
e
Sjö
gre
n
dis
eas
e
Sjö
gre
n
syn
dro
me
ski
n
basal cell
carcinoma
of cancer
of
layers of
scalded skin
syndrome
squamous cell
carcinoma of
SLE. See systemic lupus
erythematosus slipped capital
femoral epiphysis (SCPE)
small bowel series
smoking
Buer
ger
disea
se
and
emph
ysem
a and
hyper
tensio
n and
lung
cance
r and
smoking, obesity, diabetes, African-American
race/age (SODA) SODA (smoking, obesity,
diabetes, African-American race/age)
somatostatin
so
m
at
ot
ro
ph
s
sp
ec
ifi
c
gr
an
ul
es
sp
ec
tri
n
sp
er
m
at
oc
el
e
sp
er
m
at
og
en
es
is
sp
he
ro
cy
te
s
spina
bifida
occult
spinotha
lamic
pathway
spironolactone, Conn
syndrome and
splenectomy, for
hereditary spherocytes
spondylo lithiasis
spondy
lolysis
spongi
osis
spontan
eous
abortio
ns
spontaneous bacterial
peritonitis (SBP) SRY
gene
SSPE. See subacute sclerosing
panencephalitis staghorn calculi
staphylococcu
s aureus
scalded skin
syndrome
and
sialadenitis
and
staphylococcus
epidermidis
status epilepticus
STD. See sexually
transmitted disease
steeple sign
Stevens-Johnson syndrome,
drugs causing still disease
stratified
squamous
epithelium
stratum basalis
stratum
corneum
strawberry
cervix
Streptococc
us
agalactiae
Streptococc
us
pneumoniae
Streptococc
us pyogenes
Streptococc
us viridans
struma
ovarii tumor
Sturge-
Weber
syndrome
subacute
granulomatous
thyroiditis subacute
lymphocytic
thyroiditis
subacute sclerosing
panencephalitis (SSPE)
subarachnoid hemorrhage
subclones
tumors
subdural
empyema
subdural
hematoma
subendoca
rdial
infarction
sucrose
sudden infant death
syndrome (SIDS)
superficial spreading
melanoma superior
vena cava syndrome
supermale
s
ur
fa
ct
a
nt
s
ur
gi
c
al
n
e
cr
o
si
s
s
w
e
at
gl
a
n
d
s
s
w
e
at
te
st
s
yl
v
at
ic
pl
a
g
u
e
syndrome of inappropriate antidiuretic hormone
(SIADH) syngeneic graft
syphilis
c
h
a
n
c
r
e
p
e
n
i
c
il
li
n
G
a
n
d
secondary
(disseminate
d)
syringomyeli
a
systemic lupus
erythematosus (SLE)
systemic sclerosis
tabes
dorsa
lis
Taka
yasu
arteri
tis
tape
wor
m
tattoo
s,
pigm
ent
and
Tay-
Sachs
disea
se
TB.
See
tuber
culosi
s t-
cell
defici
ency
t-cell
leukemia,
virus type I
technetium
99m
TEF. See
tracheoesophageal fistula
telangiectasia
temporal (giant
cell) arteritis
tendonitis, drugs
and
ten
nis
elb
ow
ten
sio
n
hea
dac
hes
tension
pneum
othorax
teratom
as
tertia
ry
syphi
lis
teste
s
testicular atrophy
testicular
choriocarcin
oma
testicular
feminization
testicular
lymphomas
testicular
seminoma
testicular
torsion
testosterone
t
e
t
s
p
e
l
l
s
t
e
t
a
n
y
tetralogy of Fallot (TOF)
TGA. See transposition of
the great arteries theca-lutein
cysts
t
h
e
c
o
m
a
s
t
h
i
a
m
i
n
e
third-
degree
burn
thoracic
outlet
syndrome
threatened
abortion
thrombop
hlebitis
thrombotic thrombocytopenic
purpura (TTP) thunderclap headache
thymine-thymine
dimers thymoma
thymus
thyrogl
ossal
duct
cyst
thyroid
a
d
e
n
o
m
a
c
a
r
c
i
n
o
m
a
g
l
a
n
d
h
o
r
m
o
n
e
s
p
a
i
n
f
u
l
t
h
y
r
o
t
r
o
p
h
s
t
i
c
k
s
tidal volume (VT)
TIN. See
tubulointerstitial
nephritis tinea capitis
t
i
n
e
a
c
o
r
p
o
r
i
s
t
i
n
e
a
p
e
d
i
s
t
i
n
e
a
v
e
r
s
i
c
o
l
o
r
T
i
n
e
l
s
i
g
n
TLC. See total lung capacity
TNM (tumor size, lymph node involvement, metastasis)
TOF. See
tetralogy of
Fallot tonic-
clonic seizure
tonsillitis, rheumatic fever and
TORCH (toxoplasmosis, rubella, cytomegalovirus, herpes)
organisms torticollis
total
anomalous
venous return
total lung
capacity (TLC)
toxic
megacolon
toxins, hepatic necrosis and
toxoplasmosis, rubella, cytomegalovirus, herpes
(TORCH) organisms TR. See tricuspid regurgitation
tracheoesophageal fistula (TEF)
TRALI (transfusion-related
acute lung injury) transfusion-
related acute lung injury
(TRALI) transitional cell
carcinoma
transitional
epithelium
transmural
infarction
transplants,
rejection of
transposition of the great
arteries (TGA) transudate
transudative
pleural effusion
traumatic cortical
hyperostosis
Treacher Collins
syndrome
Treponema
pallidum
Trichomonas
urethritis
Trichomonas
vaginalis
trichomoniasis
Trichophyton tonsurans
tricuspid atresia
tricuspid
regurgitation
(TR)
trigeminal
neuralgia
trisomy 18
mutation
trisomy X
Trophe
ryma
whippe
lii
tropical
sprue
Trouss
eau
sign
truncus
arterios
us
TTP. See thrombotic thrombocytopenic purpura
tuberculo
sis
(TB)
diagno
sis of
granul
omas
of
immu
ne
mecha
nism
intesti
nal
location
occurrence of
secondary
tuberculous
meningitis
tuberous
sclerosis
tubulointerstitial nephritis (TIN)
tumor size, lymph node involvement,
metastasis (TNM) tumors
a
l
p
h
a
-
c
e
l
l
a
n
a
p
l
a
s
i
a
b
e
n
i
g
n
of
br
ai
n,
ch
ild
re
n
Br
en
ne
r
ca
rci
no
id
ch
ar
ac
ter
ist
ic
s
of
b
e
n
i
g
n
m
a
l
i
g
n
a
n
t
dysplasia
Eps
tein
-
Bar
r
viru
s of
fall
opi
an
tub
es
ger
m
cell
gra
nul
osa
cell
growth
depend
ence of
heart
and
i
s
l
e
t
c
e
l
l
K
r
u
k
e
n
b
e
r
g
L
e
y
d
i
g
c
e
l
l
m
o
n
o
c
l
o
n
a
l
m
y
x
o
m
a
p
a
r
o
t
i
d
g
l
a
n
d
p
h
y
l
l
o
d
e
s
p
o
l
y
c
l
o
n
a
l
s
a
l
i
v
a
r
y
g
l
a
n
d
S
e
r
t
o
l
i
c
e
l
l
Sertoli-Leydig cell
of
s
m
all
int
est
in
e
str
u
m
a
ov
ari
i
su
bc
lo
ne
s
surface
epithelial
origin
teratoma
W
a
r
t
h
i
n
y
o
l
k
s
a
c
T
ur
co
t
sy
nd
ro
m
e
T
ur
ne
r
sy
nd
ro
m
e
22
q1
1
t
y
p
h
u
s
T
z
a
n
c
k
s
m
e
a
r
Tzanck test, herpes simplex virus and
UC. See
ulcerative
colitis
ulcerative
colitis
(UC)
ulcers
d
u
o
d
e
n
a
l
g
a
s
t
r
i
c
ulnar tunnel
syndrome
upper
gastrointesti
nal series
uremia
u
r
e
t
e
r
i
c
b
u
d
u
r
e
t
h
r
a
uric
acid
stones
urinary
tract
infecti
on
urogen
ital
ridge
urolithi
asis
u
r
t
i
c
a
r
i
a
u
t
e
r
u
s
,
d
o
u
b
l
e
v
i
t
a
m
i
n
D
vitamin D-
dependent
rickets
vitamin K
vitamins
f
a
t
-
s
o
l
u
b
l
e
w
a
t
e
r
-
s
o
l
u
b
l
e
vitiligo
VMA. See
vanillylmandelic
acid volvulus
von Gierke disease
von Hippel-Lindau
(VHL) disease von
Recklinghausen
disease
von Willebrand disease
VPF. See vascular
permeability factor
VSD. See ventricular
septal defect VT. See
tidal volume
vulva
VZV. See varicella zoster virus
x
a
n
t
h
o
m
a
s
x
e
n
o
g
r
a
f
t
xeroder
ma
pigment
osa
xerosto
mia
x-
lin
ke
d
rec
ess
ive
x-
ray
s
Y
yell
ow
fev
er
vir
us
yol
k
sac
tu
mo
rs
Z
Zollinger-Ellison
zona fibrosa, myxomatous degeneration of